Vous êtes sur la page 1sur 33

Page 1 of 33

September 6, 2011 RULE 112 - PRELIMINARY INVESTIGATION AND INQUEST Pertinent Pleadings/Legal Documents: (1) Complaint-Affidavit (2) Counter-Affidavit (3) Resolution (4) Motion for Preliminary Investigation (5) Motion for Reinvestigation I. Definition, nature and purpose A. Nature and views PCGG v. Desierto Panganiban / Feb. 10 2003 FACTS 1) The Presidential Commission on Good Government (PCGG), herein petitioner, charged Disini with bribing the late President Ferdinand E. Marcos as a means to induce him to assist and favor individuals and corporate entities. The charge pertained to the negotiation, award, signing, amendment and implementation of the main and related contracts for the Philippine Nuclear Power Plant (PNPP) project of the National Power Corporation (NPC) 2) then President Marcos instructed the NPC to pursue, supervise and undertake the construction and the eventual operation of the nuclear power plant in Morong, Bataan. Because of its lack of expertise in designing and constructing commercial nuclear power plants, the NPC needed a qualified engineering firm to act as consultant to assist it in selecting a plant site, preparing equipment specifications, soliciting bids, and evaluating proposals from prospective contractors.

offense has been committed and that Disini is probably guilty thereof HELD: The Office of the Ombudsman is endowed with a wide latitude of investigatory and prosecutory prerogatives in the exercise of its power to pass upon criminal complaints. However, such authority is not absolute; it cannot be exercised arbitrarily or capriciously. Verily, the Constitution has tasked this Court to determine whether or not there has been grave abuse of discretion amounting to lack or excess of jurisdiction on the part of any branch or instrumentality of the Government, including the Office of the Ombudsman. Specifically, this Court is mandated to review and reverse the ombudsmans evaluation of the existence of probable cause, if it has been made with grave abuse of discretion. The Ombudsmans Grounds for Disinis Exoneration The ombudsman decided in favor of Disini and ordered the dismissal of the Complaint against the latter, on the basis of the following: 1. There was no testimonial evidence (a) that Disini was interested in the proposal of Westinghouse to be its special sales representative during negotiations for the award of the PNPP project, or (b) that he intervened for Burns and Roe to stop the hiring of Ebasco as NPC consultant for the project. 2. There was neither documentary evidence nor corroborating affidavit showing how Disini had actually met with Hull to assure the latter that the former could influence Marcos to overturn the award of the consulting contract to Ebasco and to eventually award the PNPP contracts to Westinghouse and to Burns & Roe. 3. Neither the Aides-Memoire allegedly sent to Marcos through Disini nor the telexes and the correspondences between the officials of Westinghouse and Burns & Roe indicated the author, the addressee, or the dates on which they were drafted or sent. 4. All the negotiations for the unlawful commissions and the actual payments thereof were based on unauthenticated documents. 5. There was no testimonial evidence that the bank transactions or the remittances questioned by the PCGG had actually been sourced from commission payments by Westinghouse and Burns & Roe. 6. There was insufficient supporting evidence for the fact that certain corporations owned or headed by Disini -- like PPI and ECCO-Asia -- were organized specifically for the PNPP project, or that Marcos had business interests in those corporations. PCGG, through its Security and Investigation Department, submitted Hulls Affidavit dated September 28, 1988. This document detailed how Hull had met and communicated with Disini to

3) Westinghouse representative approached Disini to

4)

5)

act as their go-between with Marcos. Disini was known to be the late Presidents close personal associate, whose wife was then First Lady Imelda R. Marcos first cousin and the Marcos familys personal physician. Disini relayed his acceptance of the offer to Lea Sabol, the resident agent of Westinghouse in the Philippines. Disini assured that he could influence Marcos to cause the reversal of the Decision awarding the consulting contract to Ebasco, but he also made a commitment to Hull that the former would obtain for Westinghouse the prime contract for the entire nuclear power plant project on a turn-key basis; and for Burns & Roe, the award of the main architectural and engineering subcontract for the same project. In exchange Disini is to receive $1 million After Mrs. Corazon C. Aquino took over the Presidency of the Republic, petitioner filed the previously mentioned charges against Disini before the Office of the Ombudsman which, as already stated, dismissed the charges. Hence, the instant Petition.

ISSUE WON the PCGG has submitted enough evidence sufficient to engender a well-grounded belief that an

Page 2 of 33

discuss matters leading to (1) the revocation of the consulting contract with Ebasco and (2) the eventual award of the PNPP project to Westinghouse and Burns & Roe. There was a testimony to Disinis actions. Hull stated clearly and categorically that he met with Disini and that the latter promised to wield his influence. As to the actual payment of commissions: while it may be true that the documents were not signed (for telexes are not signed), they were nonetheless identified and their sources authenticated. Often have we ruled that the validity and the merits of a party's defense or accusation, as well as the admissibility of testimonies and pieces of evidence, are better ventilated during the trial than during the preliminary investigation. Neither can the ombudsman rule on the presence or the absence of the elements of the crime, for these are by nature evidentiary and defense matters, the truth of which can be best passed upon after a full-blown trial on the merits During the preliminary investigation, the PCGG was not obliged to prove its cause beyond reasonable doubt. It would be unfair to expect the Commission to present the entire evidence needed to secure the conviction of the accused prior to the filing of the information. The reason lies in the nature and the purpose of a preliminary investigation. At this stage, the prosecutor does not decide whether the guilt of the person charged is backed by evidence beyond reasonable doubt. The former merely determines whether there is sufficient basis to believe that a crime has been committed, and whether the latter is guilty of it and should be held for trial. The established rule is that a preliminary investigation is not the occasion for the full and exhaustive display of the parties respective sets of evidence. It is for the presentation only of such evidence as may engender a well-grounded belief that an offense has been committed, and that the accused is probably guilty thereof. During the preliminary investigation, the main function of the government prosecutor -- the ombudsman in this case -- is merely to determine the existence of probable cause and, if it does exist, to file the corresponding information. Probable cause has been defined as the existence of such facts and circumstances as would excite in a reasonable mind -- acting on the facts within the prosecutors knowledge -- the belief that the person charged is probably guilty of the crime for which he or she is being prosecuted. Probable cause is a reasonable ground for presuming that a matter is or may be well-founded on such state of facts in the prosecutors mind as would lead a person of ordinary caution and prudence to believe -- or entertain an honest or strong suspicion -- that it is so. The term does not mean actual and positive cause; neither does it import absolute certainty. It is based merely on opinion and reasonable belief. Thus, a finding of probable cause

does not require an inquiry into whether there is sufficient evidence to secure a conviction. It is enough that the act or the omission complained of is believed to constitute the offense charged. Precisely, there is a trial to allow the reception of evidence for the prosecution in support of the charge. In determining probable cause, the average person weighs facts and circumstances without resorting to the calibrations of technical rules of evidence, of which such persons knowledge is nil. Rather, the lay person usually relies on the calculus of common sense, of which all reasonable persons have abundance Baytan v. COMELEC February 4, 2003 Carpio

En Banc

Facts: The Baytan brothers registered twice during the 1997 Voters Registration, once on June 15, 1997 and next on June 22, 1997 But on August 21, 1997, they wrote a letter to COMELEC informing it of its double registration and its intention of rectifying a supposedly honest mistake However, on Sept 16, 1997, COMELEC forwarded the Baytans records for evaluation; this was endorsed to the Regional Director for prosecution Atty. Ravanzo recommended filing an info for double registration which was affirmed by the Comelec en banc in its Minute Resolution Issue: 1. WON the criminal cases should be dismissed on the ground of lack of intent and substantial compliance with the requirement of cancellation of previous registration 2. WON the Comelec en bancs assumption of orig jurisdiction over the case violated the Constitution Held: 1. NO the assailed resolutions were issued in the preliminary investigation stage. A preliminary investigation is essentially inquisitorial and is only the means to discover who may be charged with a crime, its function being merely to determine probable cause. All that is required in the preliminary investigation is the determination of probable cause to justify the holding of petitioners for trial. - Also, double registration is a malum prohibitum and does not need intent for its violation 2. NO Comelec has both adjudicatory or quasi-judicial power and administrative powers and it is only when it is exercising its adjudicatory or quasijudicial powers that Comelec is mandated to hear and decide cases first by division and then, upon MR, by en banc the conduct of a preliminary investigation before the filing of an information in court does not in any

Page 3 of 33

way adjudicate with finality the rights and liabilities of the parties investigated. A preliminary investigation does not make any pronouncement as to the guilt or innocence of the party involved. Hence, a preliminary investigation cannot be considered a judicial or quasi-judicial proceeding required to be heard by the Division in the first instance. The prosecution of election law violators involves the exercise of the COMELECs administrative powers. Thus, the COMELEC en banc can directly approve the recommendation of its Law Department to file the criminal information for double registration against petitioners in the instant case. There is no constitutional requirement that the filing of the criminal information be first decided by any of the divisions of the COMELEC.

If a preliminary investigation is required before the filing of a complaint or information, the same shall be conducted by the judge of the Municipal Trial Court or the public prosecutor in accordance with the pertinent provisions of Rule 112 of the Revised Rules of Criminal Procedure. If the investigating prosecutor finds probable cause to hold the juvenile for trial, he shall prepare the corresponding resolution and information for approval by the provincial or city prosecutor, as the case may be. The juvenile, his parents/nearest relative/guardian and his counsel shall be furnished forthwith a copy of the approved resolution. Hashim vs Boncan Date: January 13, 1941 / Ponente: Laurel, J. Facts A preliminary investigation was conducted by the respondent Fiscal at which evidence was adduced warranting the filing of an information against Hashim for possessing counterfeit treasury certificates of the Commonwealth of the Philippines The information was filed in Court, and the presiding judge, upon the strength of the said preliminary investigation and sworn information, issued a warrant for the arrest of Hashim Counsel for Hashim asked the respondent fiscal to furnish the clerk of court with the testimony of the witnesses Issue Whether or not, in a preliminary investigation conducted by the fiscal, the accused is entitled to be informed of the substance of the testimony and of the evidence presented against him Held No Ratio In this case, a preliminary investigation was conducted by the respondent Fiscal at which evidence was adduced warranting the filing of an information against Hashim. The information was filed in Court, and the presiding judge, upon the strength of the said preliminary investigation and sworn information, issued a warrant for the arrest for Hashim. To ask for the abstract of testimony at this stage of the proceedings, ostensibly for no other purpose than to scrutinize the same evidence which convinced the respondent Fiscal and the presiding Judge that there was probable ground to proceed against the petitioner, is in effect, to ask for another preliminary investigation. Doctrine (kung gusto niyo lang isama sa notes) The right to a preliminary investigation is statutory, not constitutional. The purpose is to secure the innocent against hasty, malicious, and oppressive prosecutions, and to protect him from open and public accusation of crime, from the trouble, expenses and anxiety of a public trial, and also to protect the State from useless and expensive prosecutions

DOJ-NPS Manual, Part III (Preliminary Investigation), Part II (Inquest) B. Definition; when required - Rule 112, sec. 1; - DOJ-NPS Manual, Part III, secs. 1, 2, 3, 7; - RJCL, secs. 13, 8; RULE 112 - PRELIMINARY INVESTIGATION Section 1. Preliminary investigation defined; when required. Preliminary investigation is an inquiry or proceeding to determine whether there is sufficient ground to engender a well-founded belief that a crime has been committed and the respondent is probably guilty thereof, and should be held for trial. Except as provided in Section 7 of this Rule, a preliminary investigation is required to be conducted before the filing of a compliant or information for an offense where the penalty prescribed by law is at least four (4) years, two (2) months and one (1) day without regard to the fine. RJCL Sec. 8. Conduct of Initial Investigation by the Police. The police officer conducting the initial investigation of a juvenile in conflict with the law shall do so in the presence of either of the parents of the juvenile; in the absence of both parents, the guardian or the nearest relative, or a social welfare officer, and the counsel of his own choice. In their presence, the juvenile shall be informed of his constitutional rights during custodial investigation. The right of the juvenile to privacy shall be protected at all times. All measures necessary to promote this right shall be taken, including the exclusion of the media. RJCL Sec. 13. Preliminary Investigation. As far as consistent with this Rule, the preliminary investigation of a juvenile in conflict with the law shall be governed by Section 3 of Rule 112 of the Revised Rules of Criminal Procedure. If clarificatory questions become necessary, the Rule on Examination of a Child Witness shall apply.

Page 4 of 33

The investigation is advisedly called preliminary, to be followed by the trial proper. The investigating judge or prosecuting officer acts upon probable cause and reasonable belief, not upon proof beyond a reasonable doubt. The occasion is not for the full and exhaustive display of the parties' evidence; it is for the presentation of such evidence only as may engender well-grounded belief that an offense has been committed and that the accused is probably guilty thereof. When all this is fulfilled, the accused will not be permitted to cast about for fancied reasons to delay the proceedings; the time to ask for more is at the trial. Tandoc et. al. vs. Resultan et. al. July 5, 1989 / Padilla

offenses. Warrants of arrest were issued but later suspended. ISSUE: Does a city court have power and authority to conduct a new preliminary investigation of charges, which were already investigated and dismissed by the Office of the City Fiscal? YES Define preliminary investigation (PI) HELD and RATIO: A preliminary investigation is intended to protect the accused from the inconvenience, expense and burden of defending himself in a formal trial unless the reasonable probability of his guilt shall have been first ascertained in a fairly summary proceeding by a competent officer. It is also intended to protect the state from having to conduct useless and expensive trials. This is merely inquisitorial, and is often the only means to discover who can be charged with a crime. This is not a trial on the merits. Only purpose is to determine whether a crime has been committed and whether there is probable cause to believe that the accused is guilty thereof, and it does not place the person against whom it is taken in jeopardy. 2 stages in a PI: 1) the preliminary examination of the complainant and his witnesses prior to the arrest of the accused to determine whether or not there is ground to issue a warrant of arrest; 2) PI proper, wherein the accused, after his arrest, is informed of the complaint filed against him and is given access to the testimonies and evidence presented, and he is also permitted to introduce evidence in his favor (this is to determine whether or not the accused should be released or held before trial) Old crimpro rules (1964): in cases within the jurisdiction of city court, municipal court, CFI, the accused is not entitled to be heard in a PI proper. Reason: the accused will be assured of a speedy trial upon his arraignment and will lessen the period of his deprivation of liberty. You get to determine immediately whether he is probably guilty or not. Saves time and effort for everybody. HERE, the offenses charged against petitioners are within the city courts jurisdiction. So complaints can be directly filed in court. The court can do a prelim exam for purposes of issuance of warrant of arrest, then go to trial on the merits. There was NO need for the city fiscal to do a PI. The fiscals order of dismissal is NOT a bar to the direct filing of the complaints on the ground of double jeopardy. PI will NEVER give rise to the defense of double jeopardy. Double jeopardy requires a former judgment, either of acquittal or of conviction, rendered by a court competent to render the same, not only by reason of the offense committed, which must be the same or at least comprised within it, but also by reason of the place where it was committed. PI is NOT a trial. It is only to determine, before the presentation of evidence by the prosecution and by the defense, if the latter party should wish to present any, whether or not there are reasonable grounds for proceeding formally and resolutely against the accused

PARTIES: Petitioners: Pedro Tandoc, Rogelio Ercella, Rudy Diaz, Juan Rosario, Fred Menor Respondents: Judge Ricardo Resultan (San Carlos City Court), Arnulfo Payopay, Manuel Cancino, Conrado Payopay Sr. FACTS: Oct. 19, 1980 a criminal complaint was filed by the petitioners in the Office of the City Fiscal of San Carlos City, Pangasinan with several charges: Serious Physical Injuries, Slight Physical Injuries, Trespass to Dwelling against the respondents Dec. 2, 1980 - 3 of the respondents (except the judge) also filed a complaint with the same office, charging against all the petitioners the crimes of Trespass to Dwelling, Serious Oral Defamation, Grave Threats, Physical Injuries Dec. 10, 1980 the investigating fiscal found probable cause to believe that respondents did commit the crimes charged against them by petitioners and recommended that proper Informations be filed in court. Apparently, the respondents, on Oct. 19, 1980, went to the store and dining room of one Pacita Tandoc without her permission. There was a throwing of stones and hitting (as in bugbog, ok) Pacitas helpers, the petitioners. The latter went to the Brgy. Chairman and Office of the Station Commander to report the incident and give sworn statements. Respondents filed MR but denied. So the corresponding Informations were filed for the three cases in the City Court of San Carlos City. Fiscal recommended the dropping of the Dec.2 complaint filed by respondents. He said these were countercharges, because they were filed 1 month after the incident. But there was a prima facie case for the Trespass to Dwelling case filed against one of the petitioners, so an Information for that case was also filed with the same court. Respondents filed directly with the same city court these complaints against petitioners: Serious Physical Injuries, Trespass to Dwelling, Less Serious Physical Injuries, Grave Threats to Kill This city court then conducted a preliminary exam of these 4 cases and found reasonable ground to believe that petitioners may have committed these

Page 5 of 33

The crimes charged have not yet prescribed. City courts have power and authority to conduct a preliminary examination and proceed with the trial of the case properly within its jurisdiction, as long as the offense charged has not yet prescribed. HERE, petitioners cannot request for reinvestigation by the city fiscal. Reinvestigation can only be had when a case is properly under CFI jurisdiction but filed with City Court for PI purposes only, and dismissed by the CC on the ground that no prima facie case exists. But when case is cognizable by lower courts and filed therein not only for PI but trial on merits, city fiscal has no authority to reinvestigate.

Vasquez, granted but advised that "some changes be made in the information previously filed." First Information Second Information unlawfully participated in a business through the Doromal International Trading Corporation, a family corporation of which he is the President, and which company participated in the biddings conducted by the Department of Education, Culture and Sports and the National Manpower & Youth Council, which act or participation is prohibited by law and the constitution.

Quintin Doromal v. Sandiganbayan, Ombudsman, Special Prosecutor EN BANC | September 7, 1989 | GRIO-AQUINO, J. Note: Recall the jurisdiction of the Tanodbayan, Special Prosecutor, and Ombudsman FACTS: October 1987 - Special Prosecution Officer, Dionisio A. Caoili, conducted a preliminary investigation of the charge against Quintin S. Doromal, a former Commissioner of the PCGG, for violation of the AntiGraft and Corrupt Practices Act (RA 3019), Sec. 3(h), in connection with his shareholdings and position as president and director of the Doromal International Trading Corporation (DITC) which submitted bids to supply P61 million worth of electronic, electrical, automotive, mechanical and airconditioning equipment to the DECS and the National Manpower and Youth Council (or NMYC). With the approval of Special Prosecutor Raul Gonzales, Caoili filed in the Sandiganbayan an information against Doromal for having direct or indirect financial interest in DITC, an entity which transacted or entered into a business transaction or contract with DECS and the NMYC, both agencies of the government which business, contracts or transactions he is prohibited by law and the constitution from having any interest. First Case Filed Jurisdiction of the Tanodbayan Filed petition for certiorari and prohibition in the SC questioning the jurisdiction of the "Tanodbayan" to file the information without the approval of the Ombudsman after the effectivity of the 1987 Constitution SC annulled the information in accordance with the ruling in Zaldivar vs. Sandiganbayan, that the incumbent Tanodbayan (called Special Prosecutor under the 1987 Constitution and who is supposed to retain powers and duties NOT GIVEN to the Ombudsman) is clearly without authority to conduct preliminary investigations and to direct the filing of criminal cases with the Sandiganbayan, except upon orders of the Ombudsman. This right to do so was lost effective February 2, 1987. From that time, he has been divested of such authority. Upon the annulment of the information, the Special Prosecutor sought clearance from the Ombudsman to refile it, which the Ombudsman, Conrado

entered into a business transaction or contract with the Department of Education, Culture and Sports and the National Manpower and Youth Council, ... which business, contracts or transactions he [petitioner] is prohibited by law and the constitution from having any interest

Actions Taken 1. Doromal - "Motion to Quash" the information for being invalid (because there had been no preliminary investigation), and defective (because the facts alleged do not constitute the offense charged). Sandiganbayan denied the Motion. Special Prosecutor - "Motion to Suspend Accused Pendente Lite" pursuant to Section 13 R.A. 3019. This was objected by Doromal because the President had earlier approved his application for indefinite leave of absence as PCGG commissioner effective immediately and until final decision of the courts in his case. Sandigan then ordered his suspension pendente lite from his position as PCGG Commissioner and from any other office he may be holding. His MR also denied.

2.

Second Case Filed Grave Abuse of Discretion of Sandiganbayan (1) in denying Doromals Motion to Quash the information in Second Criminal Case; and, (2) in suspending him from office despite the President's having previously approved his indefinite leave of absence " until final decision" in this case. Contentions: 1. Doromal - as the preliminary investigation that was conducted prior to the filing of the original information in the Criminal Case was nullified by this Court, another preliminary investigation should have been conducted before the new information in the second case (refilled case) against him. The denial of his right to such investigation allegedly violates his right to due process and constitutes a ground to quash the information. Sandiganbayan - another preliminary investigation is unnecessary because both old and new informations involve the same subject matter a violation of Section 3 (H) of R.A. No. 3019 in relation

2.

Page 6 of 33

to Section 13, Article VII of the 1987 Constitution. Moreover, Doromal allegedly waived the second preliminary investigation by his failure to comply with the Court's Order dated August 12, 1988 directing him to submit a statement of new or additional facts, duly supported by photo copies of documents which he would present should a new preliminary investigation be ordered. Issue: Is there a need for another preliminary investigation for the refilled case against Doromal? Held: YES. But the absence of a preliminary investigation does not render the information invalid. Ratio: A new preliminary investigation of the charge against the petitioner is in order not only because the first was a nullity (a dead limb on the judicial tree which should be lopped off and wholly disregarded"-Anuran vs. Aquino) but also because the accused demands it as his right. Moreover, the charge against him had been changed, as directed by the Ombudsman. The petitioner's right to a preliminary investigation of the new charge is secured to him by Rule 112 of the 1985 Rules on Criminal Procedure. (Sec. 3 and 7)1 That right of the accused is "a substantial one." Its denial over his opposition is a "prejudicial error, in that it subjects the accused to the loss of life, liberty, or property without due process of law." Jurisprudence: The right of the accused not to be brought to trial except when remanded therefor as a result of a preliminary examination before a committing magistrate, it has been held is a substantial one. Its denial over the objections of the accused is prejudicial error in that it subjects the accused to the loss of life, liberty or property without due process of law.
1

SEC. 3. Procedure. ... no complaint or information for an offense cognizable by the Regional Trial Court shall be filed without a preliminary investigation having been first conducted. ..... SEC. 7. When accused lawfully arrested without warrant.- When a person is lawfully arrested without a warrant for an offense cognizable by the Regional Trial Court, the complaint or information may be filed by the offended party, peace officer or fiscal without a preliminary investigation having been first conducted; on the basis of the affidavit of the offended party or arresting officer or person. However, before the filing of such complaint or information, the person arrested may ask for a preliminary investigation by a proper officer in accordance with this Rules .... If the case has been filed in court without a preliminary investigation having been first conducted, the accused may within five (5) days from the time he learns of the filing of the information, ask for a preliminary investigation with the same right to adduce evidence in his favor in the manner prescribed in this Rule.

(Conde vs. Judge of Court of First Instance of Tayabas) The absence of a preliminary investigation if it is not waived may amount to a denial of due process. (San Diego vs. Hernandez) xxx the preliminary investigation in criminal cases is not a creation of the Constitution; its origin is statutory and it exists and the right thereto can be invoked when so established and granted by law. (Mariano Marcos, et al. vs. Roman A. Cruz). OSG: The right to a preliminary investigation may be waived and was in fact waived by Doromal. SC HELD: No. Since the right belongs to the accused, he alone may waive it. If he demands it, the State may not withhold it. Absence of Preliminary Investigation is not a ground to Quash Complaint or Information: Absence of a preliminary investigation is not a ground to quash the complaint or information (Sec. 3, Rule 117, Rules of Court), the proceedings upon such information in the Sandiganbayan should be held in abeyance and the case should be remanded to the office of the Ombudsman for him or the Special Prosecutor to conduct a preliminary investigation. It does not affect the courts jurisdiction over the case, nor does it impair the validity of the information or otherwise render it defective. Doromal clearly has to face the charges based on Section 13, Article VII of the 1987 Constitution providing that "the President, Vice-President, the members of the Cabinet and their deputies or assistants shall not... during (their) tenure, ...directly or indirectly... participate in any business." His suspension from office, is likewise supported by Section 13 of RA 30192 His approved leave of absence is not a bar to his preventive suspension for, as indicated by the Solicitor General, an approved leave, whether it be for a fixed or indefinite period, may be cancelled or shortened at will by the incumbent. However, he may not be suspended indefinitely as that will run counter to his rights to due process of law. Doromals preventive suspension has exceeded the reasonable maximum period of ninety (90) days provided in Section 42 of the Civil Service Decree of the Philippines (P.D. 807), being such, it should now be lifted.

Disposition: Petition for certiorari and prohibition granted. Sandiganbayan ordered to immediately remand the Criminal Case to the Office of the
2

SEC. 13. Suspension and loss of benefits .-Any public officer against whom any criminal prosecution under a valid information under this Act or under the provisions of the Revised Penal Code on bribery is pending in court, shall be suspended from office. Should he be convicted by final judgment, he shall lose all retirement or gratuity benefits under any law, but if he is acquitted, he shall be entitled to reinstatement and to the salaries and benefits which he failed to receive during suspension, unless in the meantime administrative proceedings have been filed against him.

Page 7 of 33

Ombudsman for preliminary investigation and to hold in abeyance the proceedings before it pending the result of such investigation. Doromals preventive suspension is lifted. COJUANGCO V. PCGG October 2, 1990 | J. Gancayco FACTS: In Nov. 28, 1989, Pres. Cory Aquino directed the Sol Gen to prosecute all person involved in the misuse of coconut levy funds Sol Gen created a task force to conduct a thorough study of the possible involvement of all persons filed 2 criminal complaints with PCGG (IS Nos. 74 and 75) PCGG assigned both complaints to prosecutor del Rosario for preliminary investigation (PI) In the scheduled PI, Cojuangco appeared thru his counsel and moved to disqualify/inhibit the PCGG or dismiss and alleges that the PCGG may not conduct a PI without violating Cojuangcos rights to due process and equal protection of the law and that PCGG has no right to conduct such PI these were denied 2 informations (criminal cases) were filed by PCGG with the Sandiganbayan SC directed that the Ombudsman be impleaded as party-respondent and was required to comment ISSUE: (1) WON PCGG can conduct a PI of the criminal complaints filed against them by the SolGen (2) WON PI by PCGG of the complaints violates the right of Cojuangco to due process and to equal protection of law HELD: (1) Yes.but.. (2) Yes. PCGG cannot conduct the PI with cold neutrality of an impartial judge RATIO: (1) PCGG has power to conduct PI The exclusive jurisdiction of the Tanodbayan to conduct PI of graft and corruption/unexplained wealth cases was modified when Pres. Aquino issued EO 1 creating the PCGG and constituting its membership to assist the President in the recovery of ill gotten wealth accumulated by the former President, his relatives and cronies Sec 2. The commission shall be charged with the task of assisting the President in regard to the following matters: (b) the investigation of such cases of graft and corruption as the President may assign to the Commission from time to time Sec 3. The commission shall have the power and authority: (a) to conduct investigations as may be necessary in order to accomplish and carry out the purposes of this order EO 14 also empowers the OSG and PCGG to file and prosecute all cases investigated by it under EO 1 The authority to investigate extended to the PCGG includes the authority to conduct a PI

Upon the adoption of the 1987 Consti creation of the office of the Ombudsman Art 13 Sec 1 vesting on the Ombudsman the right and the power to investigate on its own or on complaint, any act or omission of any public official, employee, office or agency which appears to be illegal, unjust, improper, or inefficient general power of investigation covers the lesser power to conduct a PI. How to reconcile? Consti provision does not show that the power of investigation including PI vested on the Ombudsman is exclusive. The Consti did not repeal or remove the power to conduct an investigation vested on the PCGG by EO 1 and 14.

(2) PI by PCGG violates Cojuangcos right to due process and equal protection of the laws Rule 112 Sec 1 Preliminary investigation an inquiry or proceeding for the purpose of determining whether there is sufficient ground to engender a well-founded belief that a crime cognizable by the RTC has been committed and that the respondent is probably guilty thereof, and should be held for trial Purpose to secure the innocent against hasty, malicious and oppressive prosecution, and to protect him from an open and public accusation of a crime, from the trouble, expense, anxiety of a public trial, and also to protect the state from useless and expensive trials Indispensable requirement of due process is that the person who presides and decides over a proceeding, including PI, must possess the cold neutrality of an impartial judge Would it be fair and just for the PCGG to conduct the PI? NO General power of investigation vested in PCGG Criminal investigation fact-finding inquiring usually conducted by law enforcement agents where they gather evidence and interview witnesses after which they assess the evidence and if they find sufficient basis, file the complaint for the purpose of PI PI ascertain if there is sufficient evidence to bring a person to trial SC says, with the PCGG having gathered the evidence and filed the complaint as a law enforcer, he cannot be expected to handle with impartiality the PI of his own complaint as a public prosecutor PCGG cannot conduct the PI of said criminal complaints with the cold neutrality of an impartial judge as it has prejudged the matter J. GUTIERREZ, CONCURRING He feels that the SC should have gone further PCGG should no longer continue conducting PI. It should limit itself to the preparation and filing of civil cases The court should have explored further on the issue of the equal protection PCGG appointment, tenure, functions, and objectives prevent it from being fair and

Page 8 of 33

objective a temporary office given a fixed mission Ombudsman created by the Constitution. Vested with independent powers and enjoys fiscal autonomy Impartiality and fundamental fairness are inherent rights of all persons brought before our criminal justice system Webb vs. De Leon 23 Aug 1995 | Puno, J.

erode the credibility of Alfaro. There is no rule of law which prohibits a court from crediting part of the testimony of a witness as worthy of belief and from simultaneously rejecting other parts which the court may find incredible or dubious. The DOJ Panel evaluated supporting statements of other witnesses. It then weighed these inculpatory evidence against exculpatory evidence of petitions ruling that an alibi cannot outweigh positive identification made by a witness. A finding of probable cause needs only to rest on evidence showing that more likely than not a crime has been committed by the suspects. DOJ did not gravely abuse its discretion when it refused to call the NBI witnesses for clarificatory questions. The decision to do so is addressed to the sound discretion of the investigator alone. If the evidence on hand already yields probable cause, the investigator need not hold a clarificatory hearing. Probable cause merely implies probability of guilt. Prelim investigation is not part of trial and it is only in a trial where n accused can demand the right to confront and cross-examine his accusers to establish his innocence. There was enough evidence in this case to make it unnecessary. Petitioners also charge NBI with violating their right to discovery proceedings during their prelim investigation by suppressing the original copy of Alfaros sworn statement. DOJ still found probable cause to charge them despite the inconsistencies in light of the totality of evidence presented by the NBI. 2. The judges just personally review the initial determination of the prosecutor finding a probable cause to see if it is supported by substantial evidence. The sufficiency of the review process cannot be measured by merely counting minutes and hours. The fact that it took them a few hours does not mean they made no personal evaluation of the evidence attached to the records of the case. DOJ Panel did not conduct the prelim investigation with indecent haste. There is no violation of the petitioners right to due process and right to impartial investigation. DOJ Order No. 223 allows the filing of an Information in court after the consummation of the prelim investigation even if the accused can still exercise the right to review of the prosecutors recommendation with the Sec. of Justice. The non-inclusion of Jessica Alfaro is anchored on RA 6981 (Witness Protection Program).

Facts: NBI filed with DOJ a letter-complaint charging Webb, Gatchalian, Lejano and 6 others with the crime of rape with homicide. DOJ formed a panel of prosecutors to conduct preliminary investigation where the NBI presented 1) sworn statement of Alfaro 2) Sworn statements of 2 former Webbs housemaids 3)sworn statement of Carlos Cristobal (passenger of flight bound for NY) 4)sworn statement of Lolita Birrer 5) sworn statements of 2 Vizcondes maids and 6) sworn statements of security guard and Gatmaitan, an engineer. Autopsy reports and genital examination were also presented. Before submitting a counter-affidavit, Webb filed with DOJ a Motion for Production and Examination of Evidence and Documents for NBI to produce. Motion was granted. NBI submitted photocopies of documents. Original sworn statement of Alfaro was lost. Webb claimed during the prelim investigation that he went to the US on 1 Mar. 1991 and did not return until 27 Oct. 1992. ISSUES: The petitioners contend that: 1. DOJ Panel gravely abused its discretion when it found probable cause 2. Judges de Leon and Tolentino failed to conduct a prelim investigation before issuing warrants of arrest against them, gravely abusing their discretion 3. DOJ Panel denied their right to due process during prelim investigation 4. DOJ Panel unlawfully intruded into judicial prerogative when it failed to charge Jessica Alfaro in the information as accused HELD: 1. DOJ Panel did not abuse its discretion when it found probable cause. The purpose of a prelim investigation under Sec. 1 of Rule 112 provides that a prelim investigation should determine WON there is sufficient ground to engender a well-grounded belief that a crime cognizable by the RTC has been committed and that the respondent is probably guilty thereof, and should be held for trial. Sec. 3 of the same Rule outlines the procedure in conducting a prelim investigation. The need to find probable cause is dictated by the Bill of Rights. Petitioners attack the truthfulness of Alfaro because she described Webbs hair as semi-blond and she committed inconsistencies in her 2 sworn statements. DOJ Panel ruled that these did not

3.

4.

C. Persons authorized to conduct - Rule 112, sec. 2; - DOJ-NPS Manual, sec. 6; - RA 6770, secs. 11 (4)(a), 15; Sec. 2. Officers authorized to conduct preliminary investigations. The following may conduct preliminary investigations:

Page 9 of 33

(a) Provincial or City Prosecutors and their assistants; (b) Judges of the Municipal Trial Courts and Municipal Circuit Trial Courts; (c) National and Regional State Prosecutors; and (d) Other officers as may be authorized by law. Their authority to conduct preliminary investigations shall include all crimes cognizable by the proper court in their respective territorial jurisdictions. Sec. 11. Structural Organization. - The authority and responsibility for the exercise of the mandate of the Office of the Ombudsman and for the discharge of its powers and functions shall be vested in the Ombudsman, who shall have supervision and control of the said Office. (4) The Office of the Special Prosecutor shall, under the supervision and control and upon the authority of the Ombudsman, have the following powers: (a) To conduct preliminary investigation and prosecute criminal cases within the jurisdiction of the Sandiganbayan; (b) To enter into plea bargaining agreements; and (c) To perform such other duties assigned to it by the Ombudsman. The Special Prosecutor shall have the rank and salary of a Deputy Ombudsman. Sec. 15. Powers, Functions and Duties. - The Office of the Ombudsman shall have the following powers, functions and duties: (1) Investigate and prosecute on its own or on complaint by any person, any act or omission of any public officer or employee, office or agency, when such act or omission appears to be illegal, unjust, improper or inefficient. It has primary jurisdiction over cases cognizable by the Sandiganbayan and, in the exercise of his primary jurisdiction, it may take over, at any stage, from any investigatory agency of government, the investigation of such cases; (2) Direct, upon complaint or at its own instance, any officer or employee of the Government, or of any subdivision, agency or instrumentality thereof, as well as any government-owned or controlled corporations with original charter, to perform and expedite any act or duty required by law, or to stop, prevent, and correct any abuse or impropriety in the performance of duties;chan robles virtual law library (3) Direct the officer concerned to take appropriate action against a public officer or employee at fault or who neglects to perform an act or discharge a duty required by law, and recommend his removal, suspension, demotion, fine, censure, or prosecution, and ensure compliance therewith; or enforce its disciplinary authority as provided in Section 21of this Act: Provided, That the refusal by any officer without just cause to comply with an order of the Ombudsman to remove, suspend, demote, fine, censure, or prosecute an officer or employee who is at fault or who neglects to perform an act or discharge a duty required by law shall be a ground for disciplinary action against said officer;

(4) Direct the officer concerned, in any appropriate case, and subject to such limitations as it may provide in its rules of procedures, to furnish it with copies of documents relating to contracts or transactions entered into by his office involving the disbursement or use of public funds or properties, and report any irregularity to the Commission on Audit for appropriate action; (5) Request any government agency for assistance and information necessary in the discharge of its responsibilities, and to examine, if necessary, pertinent records and documents; (6) Publicize matters covered by its investigation of the matters mentioned in paragraphs (1), (2), (3) and (4) hereof, when circumstances so warrant and with due prudence: Provided, that the Ombudsman under its rules and regulations may determine what cases may not be made public: Provided further, That any publicity issued by the Ombudsman shall be balanced, fair and true; (7) Determine the causes of inefficiency, red tape, mismanagement, fraud, and corruption in the government and make recommendations for their elimination and the observance of high standards of ethics and efficiency; (8) Administer oaths, issue subpoena and subpoena duces tecum, and take testimony in any investigation or inquiry, including the power to examine and have access to bank accounts and records; (9) Punish for contempt in accordance with the Rules of Court and under the same procedure and with the same penalties provided therein; (10) Delegate to the Deputies, or its investigators or representatives such authority or duty as shall ensure the effective exercise or performance of the powers, functions, and duties herein or hereinafter provided; (11) Investigate and initiate the proper action for the recovery of ill-gotten and/or unexpired wealth amassed after February 25, 1986 and the prosecution of the parties involved therein. The Ombudsman shall give priority to complaints filed against high ranking government officials and/or those occupying supervisory positions, complaints involving grave offenses as well as complaints involving large sums of money and/or properties. Velasco vs Casaclang 294 SCRA 395 August 19, 1998 Ponente: Purisima Nature: Special Civil Action in the Supreme Court. Certiorari and Prohibition. Facts: COA Audit Examiners conducted a special audit of selected transactions of the AFP Logistics Command involving the procurement of a certain amount of stainless steel meat cans. (what are meat cans? I dont know.) In their Memorandum Report, the examiners stated that they found the procurement of 28,432 pieces of meat cans to be of doubtful validity. The transaction cycle from preparation to purchase orders were all completed in just one day and

Page 10 of 33

tha the tree of the four winning bidders have common incorporators. Subsequently, the audit examiners filed with the Office of the Ombudsman a Joint Affidavit-Complaint, deploring the aforesaid transactions dubbed as anomalous and highly irregular. The Deputy Ombudsman issued an Order directing the herein petitioner and her coaccused to file a counter-affidavit and other controverting evidence, failure of which shall be construed as a waiver of their right to be heard and the premilinary investigation, after which the case shall proceed accordingly. Velasco filed a motion to direct the complainant audit examiners to particularize the offenses. The Deputy Ombudsman, without a word from the COA, issued an Order stating that petitioner was being charged with a violation of certain provisions of RA 3019. Velasco then filed a Motion to Quash, saying that the audit examiners did not charge an offense. The Deputy Ombudsman denied the motion to quash, saying that AO No. 07 does not allow such a motion.

known as the Anti-Graft and Corrupt Practice Act in an information that was filed with the Sandiganbayan by the Special Prosecutor which was approved by the Deputy Tanodbayan, after a preliminary investigation. (The information states that they enforced a Writ of Execution against a Mustang car registered in the name of Leticia Acosta-Ang, despite their knowledge that the registered owner is not the judgment debtor in a Civil Case No. 4047). March 18,1987 - Antonio Uy Lim, the plaintiff in Civil Case No. 4047 filed a complaint for rescission of the sale of the car by Juanito Ang to private respondent Leticia Acosta-Ang for being allegedly in fraud of creditors. The complaint was docketed as Civil Case No. 5307. On the same day, petitioners filed a motion for reinvestigation in the Tanodbayan which was granted. April 22, 1988 - The Tanodbayan filed with the Sandiganbayan a motion to withdraw the information against petitioners which was denied. On September 1, 1988, petitioners filed a motion to suspend proceedings in the criminal case against them on the ground of the existence of a prejudicial question in Civil Case No. 5307 which was also denied. Issue: Does the denial by the Sandiganbayan of the 2 motions (withdraw the information and suspend proceedings on the ground of a prejudicial question in a pending civil action) constitute a grave abuse of discretion despite a new finding in the investigation done by the Tandobayan? Held: No. Ratio: While the public prosecutor has the sole direction and control in the prosecution of offenses, once the complaint or information is filed in court, the court thereby acquires jurisdiction over the case and all subsequent actions that may be taken by the public prosecutor in relation to the disposition of the case must be subject to the approval of the said court. In such an instance, before a re-investigation of the case may be conducted by the public prosecutor, the permission or consent of the court must be secured. And if after such reinvestigation the prosecution finds a cogent basis to withdraw the information or otherwise cause the dismissal of the case, such proposed course of action must be addressed to the sound discretion of the court. The only instance when the appellate court should stay the hand of the trial court in such cases is when it is shown that the trial court acted without jurisdiction or in excess of its jurisdiction or otherwise committed a grave abuse of discretion amounting to such lack or excess of jurisdiction. Upon reinvestigation of the criminal case by the Tanodbayan, he found evidence tending to show that the sale of said car to the complainant by Juanita Ang, the judgment debtor, was a sham

Issue: WON the conduction of a preliminary investigation is within the jurisdiction of the Deputy Ombudsman Held//Ratio: Yes. Velasco argued that pursuant to Section 11, par.4, subpar. (a) of RA 6770, the Office of the Special Prosecutor is vested with the power and authority to conduct preliminary investigation and to prosecute criminal cases falling within the jurisdiction of the Sandiganbayan. Section 2 of Rule 112 provides that such other officers as may be authorized by law may conduct preliminary investigation. Under Section 15 of RA 6770, it is provided that the Office of the Ombudsman has primary jurisdiction over cases cognizable by theSandiganbayan and... may take over, at any stage, from any investigatory agency of the Government, the investigation of such case. It is beyond cavil that the Ombudsman and his Deputies are, within, legal contemplation, other officers authorized by law to conduct preliminary investigation. As to determination of offense: Embraced in the powers of the Ombudsman is the discretionary power to define, supervise and control the methodology and procedure his office may adopt in connection with its investigative power. Conformably, the Deputy Ombudsman has the authority to decide what offense/s to charge on the basis of the evidence before him. Balgos et. al vs. Sandiganbayan G.R. No. 85590 / August 10, 1989 / En banc / Gancayco, J. Facts: April 18, 1986 Petitioners were charged with violation of Section 3(c) of Republic Act No. 3019, otherwise

Page 11 of 33

intended to defraud his creditors; that the deed of absolute sale appeared to be fictitious; that upon the execution of the judgment, the car was found in the possession of Alvin, the son of Juanita Ang, who admitted that the car belonged to his father by showing the receipt of its repair in the name of Juanita Ang. This is the basis of the motion for withdrawal of the information of the Tanodbayan. Although at the reinvestigation, the Tanodbayan was persuaded that in fact the sale of the car to Leticia Ang was fraudulent, this did not necessarily clear petitioners of the aforesaid Anti-Graft charge against them. Still the burden is on the petitioners to establish that they acted in good faith in proceeding with the execution on the car even they were presented evidence tending to show it did not belong to Juanito Ang anymore. The pending civil case for the annulment of the sale of the car to Leticia Ang (Civil Case No. 5307) is not determinative of the guilt or innocence of the petitioners for the acts allegedly committed by them in seizing the car. Even if the sale was void, it doesnt follow that the car was rightfully taken. The pending civil case for the annulment of the sale of the car to Leticia Ang (Civil Case No. 5307) is not determinative of the guilt or innocence of the petitioners for the acts allegedly committed by them in seizing the car. Even if in the civil action it is ultimately resolved that the sale was null and void, it does not necessarily follow that the seizure of the car was rightfully undertaken. Alonzo v Concepcion Date: January 17, 2005 Ponente: Puno Facts: May 10, 2003 - At a wedding party, SPO4 Alonzo, Rances, Salamat and Santos were drinking together in the same table. While waiting to be seated, Pedrito Alonzo was introduced by SPO4 Alonzo to Rances as his nephew and as the son of ex-Captain Alonzo. SPO4 Alonzo then introduced him to Salamat. Pedrito and his companions took their seats and started drinking at the table across SPO4 Alonzo's table. After some time, Pedrito stood up to urinate at the back of the house. Santos passed a bag to Salamat, and they followed Pedrito. Rances likewise followed them. A shot rang out. Salamat was seen placing a gun inside the bag as he hurriedly left. The wedding guests ran after Salamat. They saw him and Rances board a vehicle being driven by Santos. Pedrito's uncle, Jose Alonzo, sought the help of SPO4 Alonzo to chase the culprits. He refused and even disavowed any knowledge as to their identity. Jose Alonzo filed a complaint for murder against Salamat, Rances, Santos, SPO4 Alonzo and a certain Isidro Atienza. Pursuant to the preliminary investigation conducted, it was recommended that Salamat be charged with murder as principal, and Santos and Rances as accessories. With regard to SPO4 Alonzo and Isidro Atienza, the prosecutor found that no sufficient

evidence was adduced to establish their conspiracy with Salamat. Thereafter, an Information was prepared, charging Salamat as principal, and Rances and Santos as accessories, for the murder of Pedrito. No bail was recommended. December 17, 2003 - Judge Concepcion issued an order finding conspiracy among SPO4 Alonzo (mastermind), Rances (back-up of Salamat), Salamat (hired killer) and Santos (provider of gun) and directing the Office of the Provincial Prosecutor of Bulacan to amend the information, so as to include all the aforenamed persons as accused in this case, all as principals, within five (5) days from notice hereof. January 5, 2004 - SPO4 Alonzo filed his Motion for Reconsideration. Court had no authority to review and reverse the resolution of the Office of the Provincial Prosecutor or to find probable cause against him for the purpose of amending the Information. Prosecutor's resolution can only be reviewed by the Department of Justice, by the Court of Appeals or by the Supreme Court, when a case for certiorari is filed. January 12, 2004 - SPO4 Alonzo filed an Urgent Motion for Inhibitation. By issuing the aforementioned Order, Judge Concepcion has shown his prejudice against him and bias in favor of private complainant Jose Alonzo. He prayed that the case be reraffled to another judge. January 13, 2004 - Judge Concepcion issued an Order denying the Motion for Reconsideration and the Motion for Inhibition. Judge Concepcion stated that SPO4 Alonzo had no personality to file the said motions as he was not an accused in that case. He held that only the Office of the Provincial Prosecutor could question the first Order. January 16, 2004 - SPO4 Alonzo filed a verified affidavit-complaint. He accused respondent judge of: a) gross ignorance of the law; b) violation of Section 2, Article 3 of the 1987 Constitution; c) abuse of authority under Section 6, Rule 112 of the Rules of Court; d) knowingly rendering an unjust order; e) conduct unbecoming of a judge; and f) oppression and partiality, February 26, 2004 Judge Concepcion received the First Indorsement from the Office of the Court Administrator (OCA), requiring him to file his comment to the complaint within ten days from receipt thereof. March 4, 2004 Judge Concepcion filed his comment. o He claimed that while evaluating the records of the case, his curiosity was piqued as to why no bail was recommended for the three accused. He noticed that the five witnesses who testified during the preliminary investigation had consistent accounts of the incidents leading to the death of Pedrito. From these accounts, respondent concluded that SPO4 Alonzo and all the accused conspired to kill Pedrito,

Page 12 of 33

thus the Office of the Provincial Prosecutor erred when it merely charged Salamat as principal, and Rances and Santos as accessories, while complainant was exonerated. o Respondent stressed that he bade the prosecution to amend the Information "xxx without any sanction even hinted, should it fail to do so." The OCA recommended that the complaint be dismissed on the ground that the Order and the acts complained of were done by respondent in his judicial capacity and were not actuated by bad faith, dishonesty or similar motive. In addition, the proper remedy of the aggrieved party is to file a special civil action for certiorari under Rule 65 of the Rules of Court, and not an administrative complaint.

Issue: WON SC should follow recommendation of OCA WON Judge Concepcion is liable for conduct unbecoming of a judge Held: SC cannot follow recommendation of OCA. Judge Concepcion is liable for conduct unbecoming of a judge and is reprimanded. Ratio: Jude Concepcion overlooked the fact that there is a remedy where a prosecutor errs in not charging a person in an Information. The recourse is to appeal to the Secretary of Justice. By ordering the prosecutor to include complainant, Rances and Santos as principals in the Information, respondent arrogated unto himself the executive power of supervision and control over public prosecutors. His conduct is not only unbecoming of a judge; more importantly, it transgresses our Constitution. It is not a sufficient excuse for respondent to aver that he did not impose any sanction for noncompliance with his Order. In itself, his Order does violence to the principle of separation of powers enshrined in our Constitution. In a clash of views between the judge who did not investigate and the prosecutor who did, or between the fiscal and the offended party or the accused, that of the prosecutor's should normally prevail. Respondent judge also erred when he issued warrants of arrest for Rances and Santos without bail. As the Information has not yet been amended charging these two accused as principals to the crime of murder, they are still entitled, as mere accessories, to bail under Rule 114, Section 4 of the Revised Rules of Criminal Procedure. The Court notes with approval that respondent corrected this error by allowing Rances and Santos, with the recommendation of the prosecution, to post bail. For lack of evidence, respondent is exonerated of the other charges brought against him. Notes: The rules set the proper procedure for the investigation of complaints and designate the prosecutor to conduct the preliminary investigation. The function of a preliminary investigation is to determine whether there is sufficient ground to

engender a well-founded belief that a crime has been committed and the respondent is probably guilty thereof, and should be held for trial. It is through the conduct of a preliminary investigation that the prosecutor determines the existence of a prima facie case that would warrant the prosecution of a case. As a rule, courts cannot interfere with the prosecutor's discretion and control of the criminal prosecution. The reason for placing the criminal prosecution under the direction and control of the fiscal is to prevent malicious or unfounded prosecution by private persons. However, while prosecuting officers have the authority to prosecute persons shown to be guilty of a crime, they have equally the legal duty not to prosecute when after an investigation, the evidence adduced is not sufficient to establish a prima facie case. Judges should not unduly interfere with the exercise of the power to prosecute on the part of fiscals. It stands to reason then to say that in a clash of views between the judge who did not investigate and the fiscal who did, or between the fiscal and the offended party or the defendant, those of the Fiscal's should normally prevail. In this regard, he cannot ordinarily be subject to dictation. We are not to be understood as saying that criminal prosecution may not be blocked in exceptional cases. A relief in equity "may be availed of to stop a purported enforcement of a criminal law where it is necessary (a) for the orderly administration of justice; (b) to prevent the use of the strong arm of the law in an oppressive and vindictive manner; (c) to avoid multiplicity of actions; (d) to afford adequate protection to constitutional rights; and (e) in proper cases, because the statute relied upon is unconstitutional or was "held invalid.'"

D. Scope Paderanga v. Drilon Regalado | April 19, 1991 FACTS: 1) Information for multiple murder was filed in the Regional Trial Court, Gingoog City, against Felipe Galarion, Manuel Sabit, Cesar Sabit, Julito Ampo, Eddie Torion, John Doe, Peter Doe and Richard Doe, for the deaths on May 1, 1984 of Renato Bucag, his wife Melchora Bucag, and their son Renato Bucag II. 2) Only Felipe Galarion was tried and found guilty as charged. The rest of the accused remained at large. Felipe Galarion, however, escaped from detention and has not been apprehended since then 3) In an amended information filed on October 6, 1988, Felizardo Roxas, alias "Ely Roxas," "Fely Roxas" and "Lolong Roxas," was included as a co-accused. Roxas retained petitioner Paderanga as his counsel. 4) Petitioner filed, among others, an Omnibus Motion to dismiss, to Quash the Warrant of Arrest and to Nullify the Arraignment 5) In the course of the preliminary investigation, through a signed affidavit, Felizardo Roxas implicated herein petitioner(what a client/ friend eh noh?) in the commission of the crime charged. ISSUES:

Page 13 of 33

WON there is a probable cause to justify his inclusion in the case. YES WON the preliminary investigation towards him was completed. YES REASONING Preliminary investigation is generally inquisitorial, and it is often the only means of discovering the persons who may be reasonably charged with a crime, to enable the fiscal to prepare his complaint or information. It is not a trial of the case on the merits and has no purpose except that of determining whether a crime has been committed and whether there is probable cause to believe that the accused is guilty thereof, and it does not place the person against whom it is taken in jeopardy. The institution of a criminal action depends upon the sound discretion of the fiscal. He has the quasijudicial discretion to determine whether or not a criminal case should be filed in court. Hence, the general rule is that an injunction will not be granted to restrain a criminal prosecution. The ff are exceptions to the rule: a. To afford adequate protection to the constitutional rights of the accused; b. When necessary for the orderly administration of justice or to avoid oppression or multiplicity of actions; c. When there is a pre-judicial question which is sub judice; d. When the acts of the officer are without or in excess of authority; e. Where the prosecution is under an invalid law, ordinance or regulation; f. When double jeopardy is clearly apparent; g. Where the court has no jurisdiction over the offense; h. Where it is a case of persecution rather than prosecution; i. Where the charges are manifestly false and motivated by the lust for vengeance; and j. When there is clearly no prima facie case against the accused and a motion to quash on that ground has been denied. The case at bar, does not fall into the exceptions Paderanga had already filed his counter-affidavit, pursuant to the subpoena issued to him on April 17, 1989, wherein he controverted the charge against him and dismissed it as a malicious design of his political opponents and enemies to link him to the crime. We hold that this is sufficient compliance with the procedural requirement of the Rules of Court, specifically Section 3(b) of Rule 112 thereof. Besides, petitioner failed to show that the subpoena issued on April 25, 1989 involved a separate complaint charging an offense different and distinct from that charged in the complaint attached to the first subpoena issued to him earlier. The veracity and credibility of the witnesses and their testimonies are matters of defense best addressed to the trial court for its appreciation and evaluation.

Petitioner further submits that there is no prima facie evidence, or probable cause, or sufficient justification to hold him to a tedious and prolonged public trial, on the basis of the following grounds: the questioned resolution of respondent Gingoyon is full of factual misrepresentations or misapprehensions; respondent's reliance on the decision of the Regional Trial Court against Felipe Galarion suffers from constitutional and procedural infirmities considering that petitioner was not a party thereto, much less was he given any opportunity to comment on or rebut the prosecution evidence; reliance on Rogelio Hanopol's testimony is likewise "contemptible," it being merely hearsay in addition to the fact that petitioner was never given the opportunity to cross-examine Hanopol at the time he testified in court; and the affidavit of Roxas dated March 30, 1989, which is the only evidence against petitioner, has been rendered nugatory by his affidavit of retraction dated June 20, 1990. A preliminary investigation is defined as an inquiry or proceeding for the purpose of determining whether there is sufficient ground to engender a well founded belief that a crime cognizable by the Regional Trial Court has been committed and that the respondent is probably guilty thereof, and should be held for trial. The quantum of evidence now required in preliminary investigation is such evidence sufficient to "engender a well founded belief as to the fact of the commission of a crime and the respondent's probable guilt thereof. A preliminary investigation is not the occasion for the full and exhaustive display of the parties' evidence; it is for the presentation of such evidence only as may engender a wen grounded belief that an offense has been committed and that the accused is probably guilty thereof. We are in accord with the state prosecutor's findings in the case at bar that there exists prima facie evidence of petitioner's involvement in the commission of the crime, it being sufficiently supported by the evidence presented and the facts obtaining therein. It is a fundamental principle that the accused in a preliminary investigation has no right to crossexamine the witnesses which the complainant may present. Section 3, Rule 112 of the Rules of Court expressly provides that the respondent shall only have the right to submit a counter-affidavit, to examine all other evidence submitted by the complainant and, where the fiscal sets a hearing to propound clarificatory questions to the parties or their witnesses, to be afforded an opportunity to be present but without the right to examine or crossexamine. Thus, even if petitioner was not given the opportunity to cross-examine Galarion and Hanopol at the time they were presented to testify during the separate trial of the case against Galarion and Roxas, he cannot assert any legal right to crossexamine them at the preliminary investigation precisely because such right was never available to him. The admissibility or inadmissibility of said testimonies should be ventilated before the trial

Page 14 of 33

court during the trial proper and not in the preliminary investigation. The technical rules on evidence are not binding on the fiscal who has jurisdiction and control over the conduct of a preliminary investigation. If by its very nature a preliminary investigation could be waived by the accused, we find no compelling justification for a strict application of the evidentiary rules. II. Procedure in cases where preliminary investigation required A. In cases cognizable by RTC 1. Conducted by prosecutor - Rule 112, secs. 3, 4, 6, 8; - RJCL, sec. 13; Sec. 3. Procedure. The preliminary investigation shall be conducted in the following manner: (a) The complaint shall state the address of the respondent and shall be accompanied by the affidavits of the complainant and his witnesses, as well as other supporting documents to establish probable cause. They shall be in such number of copies as there are respondents, plus two (2) copies for the official file. The affidavits shall be subscribed and sworn to before any prosecutor or government official authorized to administer oath, or, in their absence or unavailability, before a notary public, each of whom must certify that he personally examined the affiants and that he is satisfied that they voluntarily executed and understood their affidavits. (b) Within ten (10) days after the filing of the complaint, the investigating officer shall either dismiss it if he finds no ground to continue with the investigation, or issue a subpoena to the respondent attaching to it a copy of the complaint and its supporting affidavits and documents. The respondent shall have the right to examine the evidence submitted by the complainant which he may not have been furnished and to copy them at his expense. If the evidence is voluminous, the complainant may be required to specify those which he intends to present against the respondent, and these shall be made available for examination or copying by the respondent at his expense. Objects as evidence need not be furnished a party but shall be made available for examination, copying, or photographing at the expense of the requesting party. (c) Within ten (10) days from receipt of the subpoena with the complaint and supporting affidavits and documents, the respondent shall submit his counter-affidavit and that of his witnesses and other supporting documents relied upon for his defense. The counter-affidavits shall be subscribed and sworn to and certified as provided in paragraph (a) of this section, with copies thereof furnished by him to the complainant. The respondent shall not be

allowed to file a motion to dismiss in lieu of a counter-affidavit. (d) If the respondent cannot be subpoenaed, or if subpoenaed, does not submit counter-affidavits within the ten (10) day period, the investigating office shall resolve the complaint based on the evidence presented by the complainant. (e) The investigating officer may set a hearing if there are facts and issues to be clarified from a party or a witness. The parties can be present at the hearing but without the right to examine or crossexamine. They may, however, submit to the investigating officer questions which may be asked to the party or witness concerned. The hearing shall be held within ten (10) days from submission of the counter-affidavits and other documents or from the expiration of the period for their submission. It shall be terminated within five (5) days. (f) Within ten (10) days after the investigation, the investigating officer shall determine whether or not there is sufficient ground to hold the respondent for trial. Sec. 4. Resolution of investigating prosecutor and its review. If the investigating prosecutor finds cause to hold the respondent for trial, he shall prepare the resolution and information. He shall certify under oath in the information that he, or as shown by the record, an authorized officer, has personally examined the complainant and his witnesses; that there is reasonable ground to believe that a crime has been committed and that the accused is probably guilty thereof; that the accused was informed of the complaint and of the evidence submitted against him; and that he was given an opportunity to submit controverting evidence. Otherwise, he shall recommend the dismissal of the complaint. Within five (5) days from his resolution, he shall forward the record of the case to the provincial or city prosecutor or chief state prosecutor, or to the Ombudsman or his deputy in cases of offenses cognizable by the Sandiganbayan in the exercise of its original jurisdiction. They shall act on the resolution within ten (10) days from their receipt thereof and shall immediately inform the parties of such action. No complaint or information may be filed or dismissed by an investigating prosecutor without the prior written authority or approval of the provincial or city prosecutor or chief state prosecutor or the Ombudsman or his deputy. Where the investigating prosecutor recommends the dismissal of the complaint but his recommendation is disapproved by the provincial or city prosecutor or chief state prosecutor or the Ombudsman or his deputy on the ground that a probable cause exists, the latter may, by himself, file the information against the respondent, or direct another assistant

Page 15 of 33

prosecutor or state prosecutor to do so without conducting another preliminary investigation. If upon petition by a proper party under such rules as the Department of Justice may prescribe or motu propio, the Secretary of Justice reverses or modifies the resolution of the provincial or city prosecutor or chief state prosecutor, he shall direct the prosecutor concerned either to file the corresponding information without conducting anther preliminary investigation, or to dismiss or move for dismissal of the complaint or information with notice to the parties. The same rule shall apply in preliminary investigations conducted by the officers of the Office of the Ombudsman. Sec. 5. Resolution of investigating judge and its review. Within ten (10) days after the preliminary investigation, the investigating judge shall transmit the resolution of the case to the provincial or city prosecutor, or to the Ombudsman or his deputy in cases of offenses cognizable by the Sandiganbayan in the exercise of its original jurisdiction, for appropriate action. The resolution shall state the findings of facts and the law supporting his action, together with the record of the case which shall include: (a) the warrant, if the arrest is by virtue of a warrant; (b) the affidavits, counter-affidavits and other supporting evidence of the parties; (c) the undertaking or bail of the accused and the order for his release; (d) the transcripts of the proceedings during the preliminary investigation; and (e) the order of cancellation of his bail bond, if the resolution is for the dismissal of the complaint. Within thirty (30) days from receipt of the records, the provincial or city prosecutor, or the Ombudsman or his deputy, as the case may be, shall review the resolution of the investigating judge on the existence of probable cause. Their ruling shall expressly and clearly state the facts and the law on which it is based and the parties shall be furnished with copies thereof. They shall order the release of an accused who is detained if no probable cause is found against him. Sec. 6. When warrant of arrest may issue. (a) By the Regional Trial Court. Within ten (10) days from the filing of the complaint or information, the judge shall personally evaluate the resolution of the prosecutor and its supporting evidence. He may immediately dismiss the case if the evidence on record clearly fails to establish probable cause. If he finds probable cause, he shall issue a warrant of arrest, or a commitment order if the accused has already been arrested pursuant to a warrant issued by the judge who conducted the preliminary investigation or when the complaint or information was filed pursuant to section 7 of this Rule. In case of doubt on the existence of probable cause, the judge may order the prosecutor to present additional evidence within five (5) days from notice and the issue must be resolved by the court within thirty (30) days from the filing of the complaint of information.chan robles virtual law library

(b) By the Municipal Trial Court. When required pursuant to the second paragraph of section of this Rule, the preliminary investigation of cases falling under the original jurisdiction of the Metropolitan Trial Court, Municipal Trial Court in Cities, Municipal Trial Court, or Municipal Circuit Trial Court may be conducted by either the judge or the prosecutor. When conducted by the prosecutor, the procedure for the issuance of a warrant of arrest by the judge shall be governed by paragraph (a) of this section. When the investigation is conducted by the judge himself, he shall follow the procedure provided in section 3 of this Rule. If his findings and recommendations are affirmed by the provincial or city prosecutor, or by the Ombudsman or his deputy, and the corresponding information is filed, he shall issue a warrant of arrest. However, without waiting for the conclusion of the investigation, the judge may issue a warrant of arrest if he finds after an examination in writing and under oath of the complainant and his witnesses in the form of searching questions and answers, that a probable cause exists and that there is a necessity of placing the respondent under immediate custody in order not to frustrate the ends of justice. (c) When warrant of arrest not necessary. A warrant of arrest shall not issue if the accused is already under detention pursuant to a warrant issued by the municipal trial court in accordance with paragraph (b) of this section, or if the complaint or information was filed pursuant to section 7 of this Rule or is for an offense penalized by fine only. The court shall them proceed in the exercise of its original jurisdiction. Sec. 7. When accused lawfully arrested without warrant. When a person is lawfully arrested without a warrant involving an offense which requires a preliminary investigation, the complaint or information may be filed by a prosecutor without need of such investigation provided an inquest has been conducted in accordance with existing rules. In the absence or unavailability of an inquest prosecutor, the complaint may be filed by the offended party or a peace officer directly with the proper court on the basis of the affidavit of the offended party or arresting officer or person. Before the complaint or information is filed, the person arrested may ask for a preliminary investigation in accordance with this Rule, but he must sign a waiver of the provision of Article 125 of the Revised Penal Code, as amended, in the presence of his counsel. Notwithstanding the waiver, he may apply for bail and the investigation must be terminated within fifteen (15) days from its inception. After the filing of the complaint or information in court without a preliminary investigation, the accused may, within five (5) days from the time he learns of its filing, ask for a preliminary investigation with the same right to adduce evidence in his defense as provided in this Rule.

Page 16 of 33

Sec. 8. Records. (a) Records supporting the information or complaint. An information or complaint filed in court shall be supported by the affidavits and counter-affidavits of the parties and their witnesses, together with the other supporting evidence and the resolution on the case. (b) Record of preliminary investigation. The record of the preliminary investigation, whether conducted by a judge or a prosecutor, shall not form part of the record of the case. However, the court, on its own initiative or on motion of any party, may order the production of the record or any of its part when necessary in the resolution of the case or any incident therein, or when it is to be introduced as an evidence in the case by the requesting party. RJCL Sec. 13. Preliminary Investigation. As far as consistent with this Rule, the preliminary investigation of a juvenile in conflict with the law shall be governed by Section 3 of Rule 112 of the Revised Rules of Criminal Procedure. If clarificatory questions become necessary, the Rule on Examination of a Child Witness shall apply. If a preliminary investigation is required before the filing of a complaint or information, the same shall be conducted by the judge of the Municipal Trial Court or the public prosecutor in accordance with the pertinent provisions of Rule 112 of the Revised Rules of Criminal Procedure. If the investigating prosecutor finds probable cause to hold the juvenile for trial, he shall prepare the corresponding resolution and information for approval by the provincial or city prosecutor, as the case may be. The juvenile, his parents/nearest relative/guardian and his counsel shall be furnished forthwith a copy of the approved resolution. Rodil v. Garcia May 13, 1981 Fernando 2nd Division Facts: Rodil was issued a warrant of arrest for murder His counsel insisted that the witnesses for the prosecution be recalled to enable him to crossexamine the on clarificatory and amplificatory matters He also prayed that Rodil be allowed bail Judge Garcia denied the request thus, this proceeding for certiorari and prohibition with preliminary injunction Issue: 1. WON the judge validly denied the prayer for bail 2. WON there was grave abuse of discretion in the refusal to allow Rodils counsel to recall prosecution witnesses Held: 1. NO The judge did not allow Rodil to be heard - What cannot be too sufficiently stressed is that the procedure to be followed in the hearing on an application for bail, while summary in character, is

not to be a mere sham or pretense. It must not be an exercise in futility. The accused is not to be denied his day in court. 2. NO, because it is not right given to an accused but a privilege that a judge may grant him An accused is not entitled to cross-examine the witnesses presented against him in the preliminary investigation before his arrest, this being a matter that depends on the sound discretion of the Judge or investigating officer concerned To follow the language of Abrera, petitioner as the accused "was not deprived of any right but was merely refused the exercise of a privilege." The Abrera decision likewise stands for this proposition first set forth in the aforecited Dequito case: "There are an infinite number of things which a party may not in strict law do or cause to be done but which may be permitted by the court in the exercise of its discretion and in the interest of justice. Specially is this true in matters affecting the conduct of the trial and the calling, recalling and examination of witnesses." "The judge is not a ministerial officer reduced to recording what takes place and what witnesses day in the examination. Above all, his is the great responsibility of safeguarding the accused from groundless or vindictive prosecution. If the justice of the peace is to ascertain, as he must, whether a crime has been committed and, if so, whether there is probable cause that the accused committed it, his authority cannot be confined as in a straight jacket to the stiffness of medieval and outmoded technicalities of practice." 13 It thus appears clearly that in the exercise of his discretion respondent Judge could have granted the request and thus avoided the necessity of this character having to be filed. The interest of a more speedy and a more efficient administration of justice would be best served if there is a greater awareness on the part of judges that in addition to safeguarding the express rights of an accused person, a matter mandated by the Constitution or the Rules of Court, they should likewise exercise their discretion in such a way that the purpose of a preliminary investigation, the avoidance of groundless or vindictive prosecutions, could be attained in as fair and objective manner as possible

(Allado vs. Diokno supra case, rule 113 first case) 2. Conducted by MTC Judge Rule 112, secs. 3, 5, 6, 8; (see above) - RJCL, sec. 13; (see above) B. In cases cognizable by MTC 1. Conducted by prosecutor - Rule 112, secs. 1 [par. 2] 3, 4, 6, 8; - RJCL, sec. 13; 2. Conducted by MTC Judge - Rule 112, secs. 3, 5, 6, 8; - RJCL, sec. 13; C. In cases cognizable by Sandiganbayan

Page 17 of 33

1. Conducted by prosecutor or MTC Judge - Rule 112, secs. 3, 4, 5, 8; 2. Conducted by Ombudsman/Special Prosecutor - RA 6770 (posted online, did anyone print this?) - Administrative Order No. 07, Rule II, secs. 1, 2, 4, 5, 6, 7; Administrative Order No. 07 Rules of Procedure of the Office of the Ombudsman Rule II: PROCEDURE IN CRIMINAL CASES Section 1. Grounds A criminal complaint may be brought for an offense in violation of R.A. 3019, as amended, R.A. 1379 as amended, R.A. 6713, Title VII, Chapter II, Section 2 of the Revised Penal Code, and for such other offenses committed by public officers and employees in relation to office. Section 2. Evaluation Upon evaluating the complaint, the investigating officer shall recommend whether it may be: a) dismissed outright for want of palpable merit; b) referred to respondent for comment; c) indorsed to the proper government office or agency which has jurisdiction over the case; d) forwarded to the appropriate office or official for factfinding investigation; e) referred for administrative adjudication; or f) subjected to a preliminary investigation. Section 3. Preliminary investigation; who may conduct. Preliminary Investigation may be conducted by any of the following: 1) Ombudsman Investigators; 2) Special Prosecuting Officers; 3) Deputized Prosecutors; 4) Investigating Officials authorized by law to conduct preliminary investigations or 5) Lawyers in the government service, so designated by the Ombudsman. Section 4. Procedure The preliminary investigation of cases falling under the jurisdiction of the Sandiganbayan and Regional Trial Courts shall be conducted in the manner prescribed in Section 3, Rule 112 of the Rules of Court, subject to the following provisions: a) If the complaint is not under oath or is based only on official reports, the investigating officer shall require the complainant or supporting witnesses to execute affidavits to substantiate the complaints. b) After such affidavits have been secured, the investigating officer shall issue an order, attaching thereto a copy of the affidavits and other supporting documents, directing the respondents to submit, within ten (10) days from receipt thereof, his counter-affidavits and controverting evidence with proof of service thereof on the complainant. The complainant may file reply affidavits within ten (10) days after service of the counter- affidavits.

c) If the respondents does not file a counter-affidavit, the investigating officer may consider the comment filed by him, if any, as his answer to the complaint. In any event, the respondent shall have access to the evidence on record. d) No motion to dismiss shall be allowed except for lack of jurisdiction. Neither may a motion for a bill of particulars be entertained. If respondents desires any matter in the complainant's affidavit to be clarified, the particularization thereof may be done at the time of clarificatory questioning in the manner provided in paragraph (f) of this section. e) If the respondents cannot be served with the order mentioned in paragraph 6 hereof, or having been served, does not comply therewith, the complaint shall be deemed submitted for resolution on the basis of the evidence on the record. f) If, after the filing of the requisite affidavits and their supporting evidences, there are facts material to the case which the investigating officer may need to be clarified on, he may conduct a clarificatory hearing during which the parties shall be afforded the opportunity to be present but without the right to examine or cross-examine the witness being questioned. Where the appearance of the parties or witnesses is impracticable, the clarificatory questioning may be conducted in writing, whereby the questions desired to be asked by the investigating officer or a party shall be reduced into writing and served on the witness concerned who shall be required to answer the same in writing and under oath. g) Upon the termination of the preliminary investigation, the investigating officer shall forward the records of the case together with his resolution to the designated authorities for their appropriate action thereon. No information may be filed and no complaint may be dismissed without the written authority or approval of the Ombudsman in cases falling within the jurisdiction of the Sandiganbayan, or of the proper Deputy Ombudsman in all other cases. Section 5. Cases falling under the jurisdiction of municipal trial courts. Cases falling under the jurisdiction of the Office of the Ombudsman which are cognizable by municipal trial courts, including those subject to the Rule on Summary Procedure may only be filed in court by information approved by the Ombudsman or the proper Deputy Ombudsman. Section 6. Notice to parties. The parties shall be served with a copy of the resolution as finally approved by the Ombudsman or by the proper Deputy Ombudsman. Section 7. Motion for reconsideration a) Only one motion for reconsideration or reinvestigation of an approved order or resolution shall be allowed, the same to be filled within five (5)

Page 18 of 33

days from notice thereof with the Office of the Ombudsman, or the proper Deputy Ombudsman as the case may be, with corresponding leave of court in cases where information has already been filed in court; b) The filing of a motion for reconsideration/reinvestigation shall not bar the filing of the corresponding information in Court on the basis of the finding of probable cause in the resolution subject of the motion. (As amended by Administrative Order No. 15, dated February 16, 2000) III. Procedure in cases not requiring a preliminary investigation A. MTC cases or those covered by summary procedure - Rule 112, sec. 9; Sec. 9. Cases not requiring a preliminary investigation nor covered by the Rule on Summary Procedure. (a) If filed with the prosecutor. If the complaint is filed directly with the prosecutor involving an offense punishable by imprisonment of less than four (4) years, two (2) months and one (1) day, the procedure outlined in section 3(a) of this Rule shall be observed. The prosecutor shall act on the complaint based on the affidavits and other supporting documents submitted by the complainant within ten (10) days from its filing. (b) If filed with the Municipal Trial Court. If the complaint or information is filed with the Municipal Trial Court or Municipal Circuit Trial Court for an offense covered by this section, the procedure in section 3 (a) of this Rule shall be observed. If within ten (10) days after the filing of the complaint or information, the judge finds no probable cause after personally evaluating the evidence, or after personally examining in writing and under oath the complainant and his witnesses in the form of searching questions and answers, he shall dismiss the same. He may, however, require the submission of additional evidence, within ten (10) days from notice, to determine further the existence of probable cause. If the judge still finds no probable cause despite the additional evidence, he shall, within ten (10) days from its submission or expiration of said period, dismiss the case. When he finds probable cause, he shall issue a warrant of arrest, or a commitment order if the accused had already been arrested, and hold him for trial. However, if the judge is satisfied that there is no necessity for placing the accused under custody, he may issue summons instead of a warrant of arrest. B. When person lawfully arrested without warrant - Rule 112, sec. 7; - DOJ Department Order No. 61 (September 21, 1993) [New Rules on Inquest]; - Rev. Pen. Code, art. 125;

- RJCL, sec. 8; Sec. 7. When accused lawfully arrested without warrant. When a person is lawfully arrested without a warrant involving an offense which requires a preliminary investigation, the complaint or information may be filed by a prosecutor without need of such investigation provided an inquest has been conducted in accordance with existing rules. In the absence or unavailability of an inquest prosecutor, the complaint may be filed by the offended party or a peace officer directly with the proper court on the basis of the affidavit of the offended party or arresting officer or person. Before the complaint or information is filed, the person arrested may ask for a preliminary investigation in accordance with this Rule, but he must sign a waiver of the provision of Article 125 of the Revised Penal Code, as amended, in the presence of his counsel. Notwithstanding the waiver, he may apply for bail and the investigation must be terminated within fifteen (15) days from its inception. After the filing of the complaint or information in court without a preliminary investigation, the accused may, within five (5) days from the time he learns of its filing, ask for a preliminary investigation with the same right to adduce evidence in his defense as provided in this Rule. RPC 125. Delay in the delivery of detained persons to the proper judicial authorities. The penalties provided in the next preceding article shall be imposed upon the public officer or employee who shall detain any person for some legal ground and shall fail to deliver such person to the proper judicial authorities within the period of; twelve (12) hours, for crimes or offenses punishable by light penalties, or their equivalent; eighteen (18) hours, for crimes or offenses punishable by correctional penalties, or their equivalent and thirtysix (36) hours, for crimes, or offenses punishable by afflictive or capital penalties, or their equivalent. In every case, the person detained shall be informed of the cause of his detention and shall be allowed upon his request, to communicate and confer at any time with his attorney or counsel. (As amended by E.O. Nos. 59 and 272, Nov. 7, 1986 and July 25, 1987, respectively). RJCL Sec. 8. Conduct of Initial Investigation by the Police. The police officer conducting the initial investigation of a juvenile in conflict with the law shall do so in the presence of either of the parents of the juvenile; in the absence of both parents, the guardian or the nearest relative, or a social welfare officer, and the counsel of his own choice. In their presence, the juvenile shall be informed of his constitutional rights during custodial investigation. The right of the juvenile to privacy shall be protected at all times. All measures necessary to promote this right shall be taken, including the exclusion of the media.

Page 19 of 33

IV. Remedies from Preliminary Investigation A. Appeal - DOJ Department Order No. 70 (July 3, 2000) [2000 NPS Rule on Appeal]; cf. DOJ-NPS Manual, Part IV (Petition for Review); - Dimatullac v. Villon, 297 SCRA 679 (1998); (missing case. See separate handout) B. Reinvestigation/Preliminary Investigation Mario Crespo v. Hon. Leodegario Mogul EN BANC | June 30, 1987 | GANCAYCO, J.: Overview: This tackles the authority of the Circuit Courts to grant/refuse motions and proceed with arraignment and trial of a case whenever there is a simultaneous petition for review of the same case at the DOJ. Timeline: Estafa case filed at the Circuit Court Crespo moved to defer arraignment due to pending petition for review in the DOJ Circuit Court Denied motion to defer, then scheduled arraignment of Crespo Crespo elevated the motion to CA CA, upon recommendation of OSG granted TRO on arraignment DOJ gives due course to case, recommending dismissal of case to Circuit Court Circuit Court rejects motion to dismiss of fiscal (DOJ) Crespo goes to CA CA dismisses petition then lifts the TRO Crespo now goes to SC. FACTS: April 18, 1977: Assistant Fiscal Proceso K. de Gala with the approval of the Provincial Fiscal filed an information for estafa against Mario Fl. Crespo in the Circuit Criminal Court of Lucena City. Circuit Criminal Court Crespo filed a Motion to Defer Arraignment - on the ground that there was a pending Petition for Review filed with the Secretary of Justice of the resolution of the Office of the Provincial Fiscal for the filing of the information. In an order, presiding judge Mogul, denied the motion, and subsequently, the MR on that motion. However, the arraignment was deferred to afford time for petitioner to elevate the matter to the appellate court. Court of Appeals Petition for certiorari and prohibition with prayer for a preliminary writ of injunction. In an order, the Court of Appeals restrained Judge Mogul from proceeding with the arraignment of the accused until further orders of the Court. OSG: recommended that the petition be given due course. CA: Granted the writ and perpetually restraining the judge from enforcing his threat to compel the arraignment of the accused in the case until the Department of Justice shall have finally resolved the petition for review.

Secretary of Justice Recommendation Justice USec, Hon.Catalino Macaraig, Jr., resolved the petition for review, reversed the resolution of the Office of the Provincial Fiscal and directed the fiscal to move for immediate dismissal of the information filed against Crespo. A Motion to Dismiss for insufficiency of evidence was filed by the Provincial Fiscal with the trial court, attaching thereto a copy of the letter of Undersecretary Macaraig, Jr. In an order, the private prosecutor was given time to file an opposition. Circuit Court Judge denied the motion and set the arraignment, insisting that the Motion to Dismiss is there to induce the trial court to resolve the innocence of the accused on evidence not before it but on that adduced before the Undersecretary of Justice - a matter which not only disregards the requirements of due process but also erodes the independence and integrity of the court. CA: Petition for certiorari, prohibition and mandamus with petition for the issuance of preliminary writ of prohibition and/or temporary restraining order in the Court of Appeals TRO issued by the CA against the threatened act of arraignment of the accused until further orders from the Court. CA dismissed the petition and lifted the restraining order; MR denied. SC: Petition for review decision of the CA; Prayer for reversal of decision and to enjoin Hon. Mogul from enforcing his threat to proceed with the arraignment and trial in the criminal case, for declaration of the information filed not valid and of no legal force and effect, for dismissal of the said case, and for declaration of Crespos obligation as purely civil. Without giving due course, the 2nd Division of the SC required Mogul, et al. to comment without filing a Motion to Dismiss OSG recommends that the petition be given due course, it being meritorious Private respondent through counsel filed reply asking that the petition be dismissed. 2nd Division resolved to transfer the case to the Court En Banc, the latter gave course to petition

Issue: May the trial court acting on a motion to dismiss a criminal case filed by the Provincial Fiscal upon instructions of the Secretary of Justice to whom the case was elevated for review, refuse to grant the motion and insist on the arraignment and trial on the merits? (Simply put May the courts disregard the recommendations of the DOJ/Fiscal to dismiss or pursue a case which are simultaneously filed in the regular courts?) YES. Court discusses the point where the authority of the fiscal ceases. Held: Whether the accused had been arraigned or not and whether it was due to a reinvestigation by the fiscal or a review by the Secretary of Justice where a

Page 20 of 33

motion to dismiss was submitted to the Court, the Court in the exercise of its discretion may grant the motion or deny it and require that the trial on the merits proceed for the proper determination of the case. Ratio: There are limits in the exercise of a Fiscals function in a criminal prosecution. Criminal Prosecution depends upon the sound discretion of the Fiscal It is a cardinal principle that criminal actions either commenced by complaint or by information shall be prosecuted under the direction and control of the fiscal. The institution of a criminal action depends upon the sound discretion of the fiscal. He may or may not file the complaint or information, follow or not follow that presented by the offended party, according to whether the evidence in his opinion, is sufficient or not to establish the guilt of the accused beyond reasonable doubt. The reason for placing the criminal prosecution under the direction and control of the fiscal is to prevent malicious or unfounded prosecution by private persons. It cannot be controlled by the complainant. Prosecuting officers under the power vested in them by law, not only have the authority but also the duty of prosecuting persons who, according to the evidence received from the complainant, are shown to be guilty of a crime committed within the jurisdiction of their office. They have equally the legal duty not to prosecute when after an investigation they become convinced that the evidence adduced is not sufficient to establish a prima facie case. Functions of a Preliminary Investigation It is through the conduct of a preliminary investigation that the fiscal determines the existence of a prima facie case that would warrant the prosecution of a case. The Courts cannot interfere with the fiscal's discretion and control of the criminal prosecution. It is not prudent or even permissible for a Court to compel the fiscal to prosecute a proceeding originally initiated by him on an information, if he finds that the evidence relied upon by him is insufficient for conviction . Neither has the Court any power to order the fiscal to prosecute or file an information within a certain period of time, since this would interfere with the fiscal's discretion and control of criminal prosecutions. Thus, a fiscal who asks for the dismissal of the case for insufficiency of evidence has authority to do so, and Courts that grant the same commit no error. The fiscal may re-investigate a case and subsequently move for the dismissal should the reinvestigation show either that the defendant is innocent or that his guilt may not be established beyond reasonable doubt. In a clash of views between the judge who did not investigate and the fiscal who did, or between the fiscal and the offended party or the defendant, those of the Fiscal's should normally prevail. On the other hand, neither an injunction, preliminary or final nor a writ of prohibition may be issued by the

courts to restrain a criminal prosecution except in the extreme case where it is necessary for the Courts to do so for the orderly administration of justice or to prevent the use of the strong arm of the law in an oppressive and vindictive manner. Limits of the Fiscals Actions Upon filing of info in the proper court, preliminary investigation is terminated; Any request for reinvestigation by the Fiscal should then be submitted to Court for proper action, i.e. permission of Court should be secured. His actions should be within the approval of the provincial or city fiscal or the chief state prosecutor as the case maybe and it maybe elevated for review to the Secretary of Justice who has the power to affirm, modify or reverse the action or opinion of the fiscal. The Secretary of Justice may direct that a motion to dismiss the case be filed in Court or otherwise, that an information be filed in Court. The preliminary investigation conducted by the fiscal for the purpose of determining whether a prima facie case exists warranting the prosecution of the accused is terminated upon the filing of the information in the proper court. The filing of said information sets in motion the criminal action against the accused in Court. Should the fiscal find it proper to conduct a reinvestigation of the case, at such stage, the permission of the Court must be secured. After such reinvestigation the finding and recommendations of the fiscal should be submitted to the Court for appropriate action. While it is true that the fiscal has the quasi judicial discretion to determine whether or not a criminal case should be filed in court or not, once the case had already been brought to Court whatever disposition the fiscal may feel should be proper in the case thereafter should be addressed for the consideration of the Court, the only qualification is that the action of the Court must not impair the substantial rights of the accused or the right of the People to due process of law. What if The Fiscal sees no reason to pursue a criminal prosecution should Court acknowledge? Court says that the role of the fiscal or prosecutor is to see that justice is done and not necessarily to secure the conviction of the person accused before the Courts. Thus, in spite of his opinion to the contrary, it is the duty of the fiscal to proceed with the presentation of evidence of the prosecution to the Court to enable the Court to arrive at its own independent judgment as to whether the accused should be convicted or acquitted. The fiscal should not shirk from the responsibility of appearing for the People of the Philippines even under such circumstances much less should he abandon the prosecution of the case leaving it to the hands of a private prosecutor for then the entire proceedings will be null and void. The least that the fiscal should do is to continue to appear for the prosecution although he may turn

Page 21 of 33

over the presentation of the evidence to the private prosecutor but still under his direction and control. RULE: Once a complaint or information is filed in Court any disposition of the case as its dismissal or the conviction or acquittal of the accused rests in the sound discretion of the Court. A fiscal cannot impose his opinion on the trial court. This is without regard on when the arraignment of the accused or on the fact that the motion was filed after a reinvestigation or upon instructions of the Secretary of Justice who reviewed the records of the investigation. SC discourages Secretary of Justice from entertaining a petition for review or appeal from the action of the fiscal, when the complaint or information has already been filed in Court to rid of instances when recommendations by its office can be disregarded by the courts. ROBERTS, JR. V. CA March 5, 1996 | J. Davide *case is 60 pages kaya mahaba din ang digest FACTS: Started when Pepsi launched its Number Fever Promotion where it announced that all holders of crowns/caps of Pepsi, Mirinda, Mountain Dew and Seven-Up bearing the winning 3-digit number will win the full amount of the prize printed on the crowns/caps which are marked with a seven-digit security code This enticed the public to buy Pepsi softdrinks TV announcement says the winning number is 349. Complainants are now claiming for the prizes but Pepsi refused to redeem/pay the said Pepsi crowns/caps Complainants filed with the City Prosecutor of QC against Roberts et al who are the officers of Pepsi. They accuse Pepsi of Estafa Violation of consumer act Violation of EO 913 (strengthening the rulemaking and adjudicatory powers of the minister of trade and industry in order to further protect consumers) Violation of Act No. 2333 entitled an act relative to untrue, deceptive and misleading advertisements Investigating prosecutor recommended filing of an information for violation of Art 318 of RPC (estafa by other deceits) and dismissing other alleged violations due to lack of evidence City Prosecutor approved the recommendation Information for estafa was approved and was docketed Roberts et al filed a motion for reconsideration saying that there was neither fraud nor deviation from or modification of the promotional rules because it had always been clearly explained to the public that for one to be entitled to the cash prize his crown must bear both the winning

number and the correct security code as they appear in the DTI list complainants failed to allege with prima facie evidence the specific overt criminal acts or omissions allegedly committed compromise agreement entered into by Pepsi is not an admission of guilt the evidence establishes that the promo was carried out with utmost good faith and without malicious intent Roberts et al filed with the DOJ a Petition for Review They also filed in the crim case motions to suspend proceedings and to hold in abeyance issuance of warrants of arrest on the ground that they had filed the petition for review with the DOJ Judge Asuncion issued an order guiding parties that they should follow the doctrine laid down in the case of Crespo v. Mogul and not by the resolution of the DOJ Judge Asuncion also denied motion to suspend proceedings etc and directed the issuance of the warrants of arrest Roberts et al filed a special civil action with CA saying that Judge Asuncion acted without or excess of jurisdiction or with grave abuse of discretion CA issued a TRO to maintain status quo because of that, Judge Asuncion issued an order postponing indefinitely the arraignment of the petitioners CA ruled that the Joint Resolution (by prosecutors) was sufficient in itself to have been relied upon by the judge to convince himself that probable cause exists and that the mere silence of the records does not give rise to an adverse inference DOJ dismissed Roberts petition to review the joint resolution Because of this, CA promulgated a decision dismissing the petition since it was mooted with the release by the DOJ of its decision

ISSUES: (1) WON Judge Asuncion committed grave abuse of discretion in denying the motions to suspend proceedings and hold in abeyance the issuance of warrants of arrest and to defer arraignment until after the petition for review filed with the DOJ have been resolved (relying on Crespo v. Mogul) (2) WON Judge Asuncion committed grave abuse of discretion in ordering the issuance of warrants of arrest without examining the records of the PI (3) WON DOJ gravely abused its discretion in dismissing the petition for review on the following bases a) the resolution of CA denying the application for a writ of preliminary injunction and b) of Judge Asuncions denial of the abovementioned motions (4) WON CA committed grave abuse of discretion a) in denying the motion for a writ of preliminary injunction solely in the ground that Judge Asuncion had already before him the Joint resolution of the investigating prosecutor when he ordered the issuance of the warrants of arrest, and b) in ultimately dismissing the petition on the ground of mootness since the DOJ had dismissed the petition for review (5) WON SC may determine the existence of probable cause either for issuance of warrants of arrest

Page 22 of 33

against Roberts, et al or for their prosecution for the crime of estafa HELD: All yes except last. RATIO: (1) How Crespo v. Mogul should be interpreted Nothing in Crespo bars the DOJ from taking cognizance of an appeal it merely advised the DOJ to as far as practicable, refrain from entertaining a petition for review or appeal from the action of the fiscal, when complaint or information has already been filed in court Secretary of Justice is only enjoined to refrain Crespo could not have intended to repeal the last paragraph of Sec 4 Rule 112 ROC which recognizes the authority of the Secretary of Justice to reverse the resolution of the provincial or city prosecutor or chief state prosecutor upon petition by a proper party It was premature for Judge Asuncion to deny the motions on the basis that the case is already pending and that to follow the opinion of the Sec of Justice may undermine the independence and integrity of the court Real and ultimate test of the independence and integrity of the court is the filing of a motion to dismiss or to withdraw the information on the basis of a resolution of the petition for review reversing the Joint Resolution of the investigating prosecutor (2) Judge should personally examine records of PI A warrant can issue only if the judge is satisfied after an examination in writing and under oath of the complainant and the witnesses in the form of searching q&as that a probable cause exists Judge is not required to personally examine the complainant and the witnesses but he is required to personally evaluate the report and supporting documents submitted by the fiscal In the case, nothing accompanied the information upon its filing with the trial court o no affidavits of the witnesses, TSN during PI, or other docs submitted in the course o when Judge Asuncion issued the warrants of arrest, he had only the information, the amended information and the joint resolution o he did not have the records or evidence supporting the prosecutors finding of probable cause o he made no specific finding of probable cause and merely directed the issuance of warrants of arrest (3) DOJ Decision SC says that the DOJ committed grave abuse of discretion when it executed on July 23 1993 a unilateral volte-face (about face) by dismissing the petition for review It dismissed the petition simply because it thought that a review of the Joint Resolution would be an exercise in futility that any further action on the part of the Department would depend on the sound discretion of the TC (4) SC disagrees with CA Disagrees when CA said that Judge Asuncion had read and relied on the Joint Resolution and that he

was convinced that probable cause exists for the issuance of the warrants of arrest against Roberts et al Nothing in the records can provide for a reasonable basis for these assumption since Judge Asuncion made no mention of the Joint Resolution and he also didnt state that he found probable cause for the issuance of warrants of arrest Majority is not persuaded with Punos dissent (duh!) that the issue of probable cause was cured when CA sustained the finding of probable cause made by Judge Asuncion after evaluating the Joint Resolution they say that it is anchored on wrong premises since the CA did not state that it either sustained Judges finding of probable cause or that they found probable cause CA just presumed that Judge Asuncion read the Joint Resolution Majority thinks that the dissenting opinion of Puno wrongly assumes that the Joint Resolution can validly serve as sufficient basis for determination of probable cause

(5) SC cannot determine probable cause The duty of determination of probable cause is not lodged in the SC Exceptions are enumerated in Brocka v. Enrile To afford adequate protection to constitutional rights of the accused For orderly administration of justice of to avoid oppression or multiplicity of suits When there is a pre-judicial question which is sub judice When the acts of the officer are without or in excess of authority Where prosecution is under an invalid law, ordinance or regulation When double jeopardy is clearly apparent Where the court has no jurisdiction over the offense Where it is a case of persecution rather than prosecution Where the charges are manifestly false and motivated by the lust for vengeance Where there is clearly no prima facie case against the accused and a motion to quash on that ground has been denied In light of the several thousand complainants in the case and thousands more in different parts of the country, if SC holds that there is probable cause, then the filing of several thousand cases in various courts throughout the country will happen (chaos!) and Pepsi management would be exposed to harassments of warrants of arrest and would have huge expenses for bailbonds and the filing of these thousands of cases will burden further the trial calendar That is why SC will not reevaluate the evidence to determine if there is probable cause Also, the records of the PI werent forwarded to SC TC and DOJ must be required to perform their duty C.J. NARVASA, SEPARATE OPINION Determination of whether or not probable cause exists to warrant the prosecution of in court of the

Page 23 of 33

accused should be consigned and entrusted to the DOJ as reviewer of the findings of the public prosecutors SC should not perform this From a pragmatic aspect undesirable also since it would increase the workload of SC There are rules and procedural mechanisms in place for the determination of probable cause and no recourse to SC should normally be allowed

J. PUNO, DISSENTING SC needs to determine existence of probable cause because of the constitutional policy of speedy adjudication of cases it started in 1992 and J. Puno said it is now 1996 and petitioners have yet to be tried in court 3 yrs of expensive litigation! Probable cause need not be based on clear and convincing evidence of guilt, neither on evidence establishing guilt beyond reasonable doubt, and definitely not on evidence establishing absolute certainty of guilt Nothing in Soliven v. Makasiar that requires prosecutors to submit to the judge the complete records of PI especially if they are voluminous this case only held that it is the personal responsibility of the judge to determine probable cause on the basis of the report and supporting docs he may require additional supporting affidavits Forwarding of complete records is not necessary when the prosecutors report is exhaustive and accurate It is the constitutional duty of the SC in criminal litigations to not only acquit the innocent after trial but to insulate, from the start, the innocent from unfounded charges Dungog vs. CA 25 March 1988 | Sarmiento, J. Facts: First Assistant Provincial Fiscal of Bohol, Angel Ucat Jr., issued a resolution finding a case for the filing of an information for estafa against Pantaleon del Rosario (alleged misappropriation of the proceeds of the sale of 24 heads of Heifer cattle under a contract of angency). Provincial Fiscal, Enrique Inting, approved it. The information was filed with RTC of Bohol and was again approved by Provincial Fiscal. However, respondent filed a Motion for Reinvestigation with the Provincial Fiscal who, acting upon the motion, reversed himself and his First Assistant Fiscal this time finding no prima facie case. Provincial fiscal filed an Omnibus Motion for Postponement of Arraignment and To Allow Withdrawal of Information. RTC denied the Motion to Withdraw Information. CA granted the motion in favor of respondent Provincial Fiscal and Pantaleon, enjoining the Presiding Judge from proceeding with the trial. ISSUE: May a trial court deny a motion submitted by the Provincial Fiscal to dismiss an information filed by him? HELD: Yes. RTC decision is upheld.

Ratio: DOCTRINE: Once a complaint or information is filed in court any disposition of the case as to dismissal or conviction or acquittal of the accused rests in the sound discretion of the court. Although the fiscal retains the direction and control of the prosecution of criminal cases even while the case is already in court, he cannot impose his opinion on the trial court. For while it is true that the fiscal has the quasi-judicial discretion to determine whether or not a criminal case should be filed in court once the case had already been brought to court, whatever disposition the fiscal may deem proper thereafter should be addressed to the court for its consideration and approval. HISTORY OF THE DOCTRINE: US vs. Valencia: After the complaint has been presented and after trial has commenced, the court not the fiscal has full control of t and can not be withdrawn by the fiscal without consent of the court. US vs. Barredo: Provincial Fiscals are not clothed with power without the consent of the court to dismiss criminal actions actually instituted and pending. Assitant Provincial Fiscal of Bataan vs. Dollete : (doctrine put to test) Court ruled that the denial of motion to dismiss implies the prosecution of the case, although not necessarily by the same fiscal who moved for dismissal, for it is embarrassing for a prosecuting attorney to be compelled to prosecute a case when he does not have the necessary evidence to secure conviction or he himself is not convinced. People vs. Pineda: (doctrine put to severe test)the question of instituting a criminal charge is one addressed to the sound discretion of the investigjating fiscal. Between clash of views between the judge who did not investigate and the fiscal who did, Fiscals should prevail. People vs. Jamisola (docrrine is threatened): Under Rule 110, Fiscal has direction and control of prosecution, he may re-nvestigate the case and subsequently move for its dismissal. Salcedo vs. Suarez: (doctrine continues to be recognized) Provincial Fiscal has the power to conduct his own investigation or reinvestigation of a case, and thereafter he may either move to dismiss the case subject to sound discretion of the judge who usually grants although he may deny the same. Mario El Crespo vs. Hon. Leodegario Mogul et. Al., and Sta. Rosa Alining Co. vs. Asst. Prov. Fiscal Zabala : power of the court to deny a motion to dismiss filed a by a prosecuting fiscal continues. Edillon vs. Narvios: when a fiscal files a motion to dismiss, it rests in the sound discretion of the judge whether to accede to such motion or not. Ordinarily, of course, he will dismiss the action in accordance with the suggestion of an experienced fiscal who has personally investigated the facts. Velasquez vs Undersecretary of Justice 182 SCRA 389 February 15, 1990 Ponente: Grio-Aquino

Page 24 of 33

Nature: Petition for certiorari to review the resolution/letter of the Undersecretary of Justice Facts: Edgardo Avila was a Cash and Business Development Consultant of the Techtrade Management International Corporation, authorized to follow-up business transactions, including loan applications submitted to the company. In 1986, he informed the company that he had a borrower (whom he did not identify) for P200,000. This was approved by the company which subsequently issued to him a check. Instead of returning the borrowed amount, Avila resigned from the company. Velasquez, Executive Vice-president/Managing Director of Techtrade filed a complaint for estafa against Avila. The Manila City Fiscal filed an information for estafa against Avila in the RTC. Before arraignment, Avila filed a petition for review in the DOJ. The Undersecretary of Justice granted the petition and directed the City Fiscal to conduct a reinvestigation of the case so as Avila may properly present evidence, etc. Issue: WON the Order of the DOJ Usec for reinvestigation was proper Held//Ratio: No. Invoking Crespo vs Mogul, the Court ruled that once the information is filed in court, the court acquires complete jurisdiction over it. The Court is the best and sole judge on what to do with the case before it. The Secretary of Justice should, as far as practicable, refrain from entertaining petition for review or appeal from the action of the fiscal when the complaint or information has already been filed in Court. The matter should be left entirely for the determination of the Court. People vs. Beriales G.R. No. L-39962 / April 7, 1976 / 2nd division / Concepcion Jr., J Facts: Ricardo Beriales Benedicto Custodio and Pablito Custodio were charged with the crime of murder in an information filed by the City Fiscal of Ormoc City. They allegedly stabbed Saturnina Porcadilla, inflicting mortal wounds which caused her death. December 3 The appellants counsel moved for reinvestigation of the case, which the court granted. The RTC postponed the hearing of the case to December 17 and 18, 1974 in view of the City Fiscal's motion "for a deferment of the hearing or trial set for December 5 and 6, 1974 until such time the reinvestigation shall have been terminated for which the result of said reinvestigation will be submitted to this Honorable Court for its resolution in the premises. December 6 The RTC, motu proprio cancelled the aforesaid hearings on December 17, and 18, 1974, and, instead, reset the arraignment and trial of the case to December 10 and 11, 1974.

December 10 During the hearing, appellants counsel manifested to the court that pursuant to its approval of his motion for reinvestigation, the City Fiscal had set the reinvestigation for December 12, 1974. The court a que still issued an order that set the hearing of the case the next day, at which hearing, counsel reiterated that said reinvestigation should first be finished and the corresponding resolution rendered thereon and submitted to the court before any trial of the case should take place. The RTC re-scheduled the hearing to December 13. December 13 - Counsel for the appellant asked the court to wait for the City Fiscal to appear, since the reinvestigation of the case had already been terminated and the Fiscal, if given a chance, might be able to report on said reinvestigation. The trial court, however, insisted in arraigning the appellants. When arraigned, the three appellants declined to plead, saying: "I am not going to answer the question because the Fiscal is not yet around." Thereupon, the trial court entered a plea of "Not Guilty" for each of them. The trial court, ordered the presentation of evidence by the private prosecutor since he had been previously authorized by the City Fiscal to handle the case. The fiscal was absent so his report on the investigation was not presented in court. When asked if the defense wanted to cross-examine the prosecutions witnesses, the appellants counsel reiterated his manifestation that they would not go to trial until the City Fiscal shall have submitted the result of the reinvestigation to the court, and the court each time ruled that it considered such manifestation as a waiver on the part of the appellants to crossexamine the witnesses. When it was the defenses turn to present evidence, appellants' counsel manifested that the appellants were not agreeing to the trial of the case unless they first received the result of the reinvestigation conducted by the City Fiscal. Nonetheless, the trial court promulgated its judgment on December 17. Issue: Were the appellants denied due process of law? Held: Yes. Ratio: After the trial court granted the appellants' motion for reinvestigation, it became incumbent upon the court to hold in abeyance the arraignment and trial of the case until the City Fiscal shall have conducted and made his report on the result of such reinvestigation. That was a matter of duty on its part, not only to be consistent with its own order but also to do justice aid at the same time to avoid a possible miscarriage of justice. The motion for reinvestigation is based upon the ground that it was Felipe Porcadilla (husband and father, respectively, of the two deceased, Saturnina Porcadilla and Quirino Porcadilla) who was the aggressor for having attacked and seriously wounded appellant Pablito Custodio, it was entirely possible for the City Fiscal to modify or change his conclusion after conducting the reinvestigation.

Page 25 of 33

The RTC committed a serious irregularity which nullifies the proceedings because such a procedure is repugnant to the due process clause of the Constitution. Also, there was was the total absence of the City Fiscal and/or any of his assistants or special counsel on December 13, 1974, when the appellants were arraigned and when the private prosecutor presented evidence and rested the case supposedly for the People. Under the ROC, all criminal actions either commenced by complaint or by information shall be prosecuted under the direction and control of the fiscal. In the trial of criminal cases, it is the duty of the public prosecutor to appeal for the government. While there is nothing in the rule of practice and procedure in criminal cases which denies the right of the fiscal, in the exercise of a sound discretion, to turn over the active conduct of the trial to a private prosecutor, nevertheless, his duty to direct and control the prosecution of criminal cases requires that he must be present during the proceedings. In the present case, although the private prosecutor had previously been authorized by the special counsel Rosario R. Polines to present the evidence for the prosecution, nevertheless, in view of the absence of the City Fiscal at the hearing on December 13, 1974, it cannot be said that the prosecution of the case was under the control of the City Fiscal. It follows that the evidence presented by the private prosecutor at said hearing could not be considered as evidence for the plaintiff, the People of the Philippines. Cf. Effects of absence of or irregularity in preliminary investigation Socrates v Sandiganbayan Date: February 20, 1996 / Ponente: Regalado Facts: This involves two consolidated original actions for certiorari and prohibition filed by petitioner Salvador P. Socrates assailing the orders and resolution issued by respondent Sandiganbayan in Criminal Cases Nos. 18027 and 18028, both entitled "People of the Philippines vs. Salvador P. Socrates." 1. In G.R. Nos. 116259-60, the validity of the informations filed in Criminal Cases Nos. 18027-28 is being contested on three grounds (1) the respondent court did not acquire jurisdiction over the case on the ground that an inordinate delay of six years between the conduct of the preliminary investigation and the subsequent filing of the informations against petitioner constitutes a violation of his constitutional rights to a speedy disposition of the case and due process of law pursuant to the Tatad doctrine; (2) the facts charged do not constitute an offense; and (3) since the acts charged in the complaints filed before the Tanodbayan are different from the charges contained in the informations, another preliminary

2.

investigation should have been conducted, in the absence of which there is a denial of due process. In G.R. Nos. 118896-97, petitioner questions the validity of the suspension order in that: (1) he may not be suspended while the issue on the validity of the informations filed against him is still pending review before the Supreme Court; and (2) Section 13 of Republic Act No. 3019, which forms the basis of the order of suspension, is unconstitutional on the ground that it constitutes an undue delegation of the authority to suspend which is essentially an executive power. In addition, petitioner again attacks the legality of the subject informations for having been filed in violation of the due process and equal protection clauses by reason of the noninclusion therein, as co-principals, of the members of the Sangguniang Panlalawigan who approved the purchase of the vessel, as well as the board of directors of ERA Technology and Resource Corporation which entered into a contract with the Province of Palawan.

Background: Socrates (P) who is the incumbent governor of Palawan, was first elected governor of the said province in 1968 and was again reelected in both the 1971 and 1980 elections, until he was replaced by private complainant Rodriguez as Officer-InCharge Governor after the EDSA Revolution in February 1986. Subsequently, both petitioner and Rodriguez ran for governor in the 1988 elections where the latter emerged victorious. In the 1992 synchronized national and local elections, the two again contested the gubernatorial post; and this time, it was petitioner who won. Meanwhile, at the time Rodriguez was still the OIC Governor of the province, the Provincial Government of Palawan, as represented by Rodriguez and the Provincial Board Members of Palawan, filed before the Office of the Tanodbayan two (2) complaints both dated December 5, 1986 and docketed as TBP No. 86-01119. The first complaint charged petitioner with violation of Section 3(b) of Republic Act No. 3019 (the Anti-Graft and Corrupt Practices Act) and the second charged petitioner, together with several other provincial officers, with violation of Section 3(a) and (g) of the same law Issues: 1. WON there was a violation of Socrates constitutional right to a speedy trial WON allegations in information constitute an offense WON information should be declared null and void (Socrates asserted that the elements of the offenses charged in the complaints are different from those stated in the informations which were filed before the Sandiganbayan, and that since there was no preliminary investigation conducted with respect to the latter, such informations should be declared null and void for lack of due process.) 2. WON preventive suspension is legal Held & Ratio: 1.

Page 26 of 33

No. In the present case, as distinguished from the factual milieu obtaining in Tatad v Sandiganbayan, respondent court found that the six-year delay in the termination of the preliminary investigation was caused by petitioner's own acts (i.e. delaying tactics). Furthermore, the allegation that the filing of the complaint was politically motivated does not serve to justify the nullification of the informations where the existence of such motive has not been sufficiently established nor substantial evidence presented in support thereof. Yes. Socrates stands charged with a violation of Section 3(h) for intervening in his official capacity as Governor of Palawan in reviewing and approving the disbursement voucher dated August 2, 1982 for payment in favor of ERA Technology Resources Corporation where he was one of the incorporators and members of the board of directors. Such allegation clearly indicates the nature and extent of petitioner's participation in the questioned transaction. Without petitioner's approval, payment could not possibly have been effected. SC does not find any flaw in the information filed in Criminal Case No. 18028, for violation of Section 3(e), which would warrant the dismissal thereof. Evidentiary facts need not be alleged in the information because these are matters of defense. Informations need only state the ultimate facts; the reasons therefor could be proved during the trial. No. SC has repeatedly held that when the facts, acts and circumstances are set forth in the body of an information with sufficient certainty to constitute an offense and to apprise the defendant of the nature of the charge against him, a misnomer or innocuous designation of a crime in the caption or other parts of the information will not vitiate it. In such a case, the facts set forth in the charge controls the erroneous designation of the offense and the accused stands indicted for the offense charged in the statement of facts. The erroneous designation may be disregarded as surplusage. Furthermore, it will be observed that it is the same section of the law which is involved in the present case, that is, Section 3 of Republic Act No. 3019, albeit it defines several modes of committing the same offense. It is an old and well-settled rule in the appreciation of indictments that where an offense may be committed in any of several different modes, and the offense, in any particular instance, is alleged to have been committed in two or more of the modes specified, it is sufficient to prove the offense committed through any one of them, provided that it be such as to constitute the substantive offense. Thereafter, a judgment of conviction must be sustained if it appears from the evidence in the record that the accused was guilty as charged of any one of these modes of the offense. Neither will the absence of a preliminary investigation, assuming that it is necessary to conduct a new one, affect the validity of the informations filed against petitioner. It has been consistently held that the absence of a preliminary investigation does not impair the validity of the criminal information or render it defective. Dismissal of the case is not the remedy. It is not a ground for

the quashal of a complaint or information. The proper course of action that should be taken is for the Sandiganbayan to hold in abeyance the proceedings upon such information and to remand the case to the office of the Ombudsman for him or the Special Prosecutor to conduct a preliminary investigation if the accused actually makes out a case justifying such relief. 2. Yes. SC has ruled that under Section 13 of the anti-graft law, the suspension of a public officer is mandatory after the validity of the information has been upheld in a pre-suspension hearing conducted for that purpose. This pre-suspension hearing is conducted to determine basically the validity of the information, from which the court can have a basis to either suspend the accused and proceed with the trial on the merits of the case, or correct any part of the proceeding which impairs its validity. Once the information is found to be sufficient in form and substance, then the court must issue the order of suspension as a matter of course. There are no ifs and buts about it. This is because a preventive suspension is not a penalty. It is not imposed as a result of judicial proceedings. It has equally been ruled that the failure of the fiscal to include the other public officials who appear to be responsible for the offense charged as co-accused in the information filed against the accused does not in any way vitiate the validity of the information. Section 1, Rule 110 of the Rules of Court, as reformulated in Section 2, Rule 110 of the 1985 Rules on Criminal Procedure What the rule demands is that all persons who appear responsible shall be charged in the information, which conversely implies that those against whom no sufficient evidence of guilt exists are not required to be included. A failure to include other persons who appear to be responsible for the crime charged is not one of the grounds provided under Section 3, Rule 117 for which a motion to quash the information against the accused may be filed. Granting arguendo that this plaint of petitioner may be invoked as a ground for the quashal of the informations, the motion to quash must still be denied for having been filed only after petitioner had been arraigned. It is significant and demonstrative of petitioner's strategy that from the inception of the criminal complaint before the Ombudsman and during the conduct of the preliminary investigation, until the filing of the informations before the Sandiganbayan and up to the denial of his amended and consolidated motion to quash, herein petitioner has not been heard to complain about the alleged noninclusion of the other supposed offenders. Indeed, it is now much too late for petitioner to invoke and exploit this particular unfounded issue. People v. Nicha Melo | January 19, 1995 Facts:

Page 27 of 33

October 1990, at around 7 o'clock in the evening, Jojo Belmonte went out of his house to buy some cigarettes in a nearby store. (Before Jojo Belmonte could buy the cigarettes, Doro Nitcha arrived, uttered the words "You are one of them" ("Maysa ca met") in the vernacular, then started mauling him. Jojo fought back. A few minutes had gone into the fight when May Villarica (a.k.a. Lydia) Joselito, Agustin and Marcelina (Nenet), all surnamed Sibayan, arrived. May and Joselito tried to pacify the two protagonists, however, their efforts proved futile as Doro Nitcha refused to be pacified. The fighting stopped upon the arrival of Doro's sister Victoria Corpuz. Likewise, Marcelina (Nenet), Agustin, May and Joselito proceeded towards their house Not long thereafter and while the Sibayans were still on their way, appellant Florestan Nitcha, brother of Doro, arrived at the sari-sari store brandishing a gun and shouting in Tagalog, "Walanghiya kayo, putangina ninyo, papatayin ko kayong lahat!" After uttering those words, appellant fired his gun in the direction of the Sibayans, the bullet hitting May at the back of her head and existing through the middle of her forehead. Appellant then aimed his gun at Joselito but missed. May died from the gunshot wound. Appellant eventually surrendered himself. At the trial, he was found guilty . Accused-appelant claims his innocence and it was a companion of Jojo Belmonte who fired the gun. ISSUE WON the accused if guilty beyond reasonable doubt. YES Ratio JOJOs COMPANION The SC did not believe that another person pulled Nitchas gun. Three witnesses attest that there was no such person and that the accused was the one who shot the victim. The fact that during the trial, Jojo was unable to convey the number of seconds in a minute, is of no moment. The basic faux pas from the witness can hardly diminish his veracity of the entire narration of how the felony was perpetrated for the simple, nay, obvious reason that an omission of this character is insignificant to merit rejection of the whole testimony, unflawed as it has remained. Evidence and clear testimony points out that the accused indeed committed the crimes (best if you read transcript sa mismong kaso) ON Effects of absence of or irregularity in preliminary investigation SC says: even on the assumption that accusedappellant's arrest was illegal for want of preliminary investigation, such a hypothesis was nonetheless negated by accused-appellant's act of posting a bail bond thereafter, apart from the fact that he entered

a plea of not guilty which is tantamount to foregoing the right to question the assumed irregularity C. Injunction and writs of restraint in proper cases Primicias v. Municipality of Urdaneta, Pangasinan October 18, 1979 De Castro En Banc Facts: Primicias was driving his car w/in the jurisdiction of Urdaneta when a member of Urdanetas Municipal Police asked him to stop He was told that he was stopped for violating Municipal Ordinance No. 3, Series of 1964 The incident took place about 200m away from a school building A criminal complaint was filed against Primicias for the violation of the ordinance. Because of this, Primicias instituted an action for the annulment of the ordinance with prayer for the issuance of preliminary injunction for the purpose of restraining Urdaneta mayor and police from enforcing the said ordinance. The writ was issued and the judge in the criminal complaint was enjoined from proceeding in the case CFI held that the ordinance was null and void and have been repealed by RA 4136 (Land Transportation and Traffic Code) Issues: 1. WON the ordinance is null and void (wont be discussed thoroughly) 2. WON the issuance of a writ of injunction was proper Held: 1. YES, it contravened the statuteRA 4136 and suffered from vagueness 2. YES - the general rule is that "ordinarily, criminal prosecution may not be blocked by court prohibition or injunction." 17 Exceptions however are allowed in the following instances: 1. for the orderly administration of justice; 2. to prevent the use of the strong arm of the law in an oppressive and vindictive manner; 3. to avoid multiplicity of actions; 4. to afford adequate protection to constitutional rights; 5. in proper cases, because the statute relied upon is unconstitutional or was held invalid. The local statute or ordinance at bar being invalid, the exception just cited obtains in this case. Hence, the lower court did not err in issuing the writ of injunction against defendants. Moreover, considering that "our law on municipal corporations is in principle patterned after that of the United States, " it would not be amiss for Us to adopt in this instance the ruling that to enjoin the enforcement of a void ordinance, "injunction has frequently been sustained in order to prevent a multiplicity of prosecutions under it."

Page 28 of 33

Hernandez vs Albano Date: January 25, 1967| Ponente: Sanchez, J. Facts The case has its roots in a complaint lodged with the Office of the City Fiscal of Manila, by Delfin Albano, Congressman for the lone district of Isabela, against Jaime Hernandez, then the Secretary of Finance and Presiding Officer of the Monetary Board of the Central Bank for violation of Article 216 of the Revised Penal Code, Commonwealth Act 6261 or Republic Act 265. At the joint investigation of the foregoing charges, Albano moved to exclude therefrom the alleged violation of Article 216 of the Revised Penal Code because the applicability of this statute was still in issue then in another case Hernandez sought the dismissal of the remaining charges upon the averment that (a) violation of Article VII, Section 11, subsection (2) of the Constitution, punishable under Commonwealth Act 626, should be prosecuted at the domicile of the private enterprises affected there by; and that (b) violation of Section 13 of Republic Act 265 is not criminal in nature To restrain the respondent Fiscals from continuing the investigation, Jaime Hernandez went to the Court of First Instance of Manila on certiorari and prohibition with a prayer for preliminary injunction

prohibited interests in corporations domiciled in Naga City (Rural Bank of Nueva Caceres, University of Nueva Caceres and Bicol Electric Co.,) and in Mandaluyong, Rizal (DMG Inc.); and that the place where the crime is to be prosecuted is "the situs of such shares." The court however held that the charges are not directed against the corporations. The necessary ingredient in the crime is the fact that Hernandez was the secretary of finance and that he allegedly was financially interested in the corporations and his office was in Manila. Court said that since criminal action must be instituted and tried in the place where the crime or an essential ingredient thereof, took place, it stands to reason to say that the Manila under the facts obtained here, have jurisdiction to investigate the violation complained of. Moreover, violation of Section 13 of Republic Act 265 is criminal in nature as clearly stated under Section 34 of the same Republic Act 2653.

Guingona et. al. vs. City Fiscal of Manila et.al. April 4, 1984 / Acting CJ Makasiar Parties: Petitioners: Teofisto Guingona (NSLA president), Antonio Martin (NLSA EVP), Teresita Santos (NSLA Gen.Manager) Respondents: Jose Flaminiano (City Fiscal of Manila), Asst. City Fiscal Felizardo Lota, Clement David (depositor) FACTS: (simplified) David invested (allegedly upon inducement) with the Nation Savings and Loan Association (NSLA) these totaled amounts: P1.275million and US$ 75,000. Then NSLA was placed under receivership by Central Bank. David learned from Central Bank that only P300,00++ of those deposits were recorded. So he filed case for estafa and violation of CB Circular No. 364 and related Central Bank regulations on foreign exchange transactions against petitioners and other NSLA directors. Petitioners in their counter-affidavits said NSLA needed funds and they put Davids accounts in a special account with greater interest and in secret documents to prevent David from being taxed fully, and Guingona and Martin already assumed portions of NSLAs liability to David in a joint promissory note (covering P1.336 million and US$75,000.00) and Guingona already paid P200,000 to David.
3

Issue Whether or not the prosecuting arm of the City of Manila should be restrained from proceeding with the investigation of the charges levelled against Jaime Hernandez

Held No

Ratio Ordinarily, criminal prosecution may not be blocked by court prohibition or injunction. However, extreme cases may, and actually do, exist where relief in equity may be availed of to stop a purported enforcement of a criminal law where it is necessary (a) for the orderly administration of justice; (b) to prevent the use of the strong arm of the law in an oppressive and vindictive manner; (c) to avoid multiplicity of actions; (d) to afford adequate protection to constitutional rights; and (e) in proper cases, because the statute relied upon is unconstitutional, or was "held invalid. Hernandezs claim is that except for his holdings in Manila's University of the East the Manila Fiscals are powerless to investigate him. His reason is that the essence of the crime is his possession of

SEC. 34. Proceedings upon violation of laws and regulations. Whenever any person or entity willfully violates this Act or any order, instruction, rule or regulation legally issued by the Monetary Board, the person or persons responsible for such violation shall be punished by a fine of not more than twenty thousand pesos and by imprisonment of not more than five years. ...

Page 29 of 33

So PI was conducted against petitioners, after David filed for these 2 crimes with the City FIscal. Petitioners moved to dismiss charges for lack of jurisdiction. They said Davids claims were purely civil in nature and were already novated. They filed petition for prohibition and injunction with a prayer for immediate issuance of restraining order and/or writ of preliminary injunction SC issued TRO after 5 days, restraining respondents to from proceeding with this PI. David filed motion to lift restraining order but was denied.

ISSUES: Does the City Fiscal have jurisdiction to investigate the charges of estafa and violation of CB Circ. No. 364? (NO, because they are wrongly filed. Case here is not criminal in nature) Was the petition for prohibition and injunction proper here? (YES. City Fiscal has no jurisdiction) HELD and RATIO: NCC 1980 says Davids investments are contracts of simple loan (a.k.a. mutuum), not contracts of deposit. So David and NSLA are creditor and debtor respectively. Bank (I guess NSLA was a bank) has obligation to return the amount deposited, NOT the same exact money deposited. Banks failure to do so WILL NOT constitute RPC 315 par.1(b) estafa through misappropriation. It will give rise to civil liability, and city fiscal has no jurisdiction over it. One of RPC estafa elements is the obligation to deliver or return the SAME money, goods or personal property that he received. Eh you cant do that here, because these deposits are loans. In simple loan, the borrower acquires ownership over the thing or money received and he can dispose of the same and it is NOT misappropriation. Even if we allow that NSLAs failure to return to David his money is RPC estafa, still, any criminal liability is avoided since Guingona and Martin already assumed NSLAs obligation to David. The banks contract with David (now a deposit contract) was novated. Guingona and Martin became the debtors, David became creditor. Novation does not extinguish criminal liability but it prevents rise of criminal liability as long as it is done before the filing of criminal information in court. Complainant is now stopped from insisting that the original obligation be enforced. (Gonzales vs. Serrano and Ong vs CA, both citing People vs. Nery) HERE, there was indeed novation before the filing of Information. Guingona and Martin already assumed NSLAs obligation. So no criminal liability but theres civil liability on these 2 to pay David. City fiscal has no jurisdiction over a purely civil liability. So they must be restrained from proceeding with the criminal case. GENERAL RULE: the prosecution in a criminal offense cannot be the subject of prohibition and injunction EXCEPTIONS: (i.e. can resort to the extraordinary writs of prohibition and injunction in extreme cases, like... Primicias vs. Municipality of Urdaneta, Pangasinan, citing Ramos vs. Torres and Hernandez vs. Albano) for the orderly administration of justice;

to prevent the use of the strong arm of the law in an oppressive and vindictive manner; to avoid multiplicity of actions; to afford adequate protection to constitutional rights; in proper cases, because the statute relied upon is unconstitutional or was held invalid Lopez vs. The City Judge: certiorari and prohibition, as extraordinary legal remedies, are in the ultimate analysis, intended to annul void proceedings; to prevent the unlawful and oppressive exercise of legal authority and to provide for a fair and orderly administration of justice. On CB Circ 364 violation: this circular requires CB approval before depositing foreign currency in banks. This wasnt violated because it can be assumed (and indeed it happened) that Davids dollar deposits were first converted into pesos before deposit with NSLA.

D. Bail - Rule 114, sec. 26; - DOJ-NPS Manual, Part V, secs. 1-13; Sec. 26. Bail not a bar to objections on illegal arrest, lack of or irregular preliminary investigation. An application for or admission to bail shall not bar the accused from challenging the validity of his arrest or the legality of the warrant issued therefore, or from assailing the regularity or questioning the absence of a preliminary investigation of the charge against him, provided that he raises them before entering his plea. The court shall resolve the matter as early as practicable but not later than the start of the trial of the case. Rolito Go v. CA, Hon. Benjamin Pelayo, People of the Philippines EN BANC | February 11, 1992 | FELICIANO, J. FACTS: Eldon Maguan was driving his car along Wilson St., San Juan, Metro Manila, heading towards P. Guevarra St. Rolito Go entered Wilson St., where it is a one-way street and started travelling in the opposite or "wrong" direction. At the corner of Wilson and J. Abad Santos Sts., Rolitos and Maguan's cars nearly bumped each other. Rolito alighted from his car, walked over and shot Maguan inside his car. Rolito then boarded his car and left the scene. A security guard at a nearby restaurant was able to take down Rolitos car plate number. The police arrived shortly thereafter at the scene of the shooting and there retrieved an empty shell and one round of live ammunition for a 9 mm caliber pistol. Verification at the LTO showed that the car was registered to one Elsa Ang Go. The following day, the police returned to the scene of the shooting to find out where the suspect had come from; they were informed that Rolito Go had dined at Cravings Bake Shop shortly before the shooting. The police obtained a facsimile or impression of the credit card used by petitioner

Page 30 of 33

from the cashier of the bake shop. The security guard of the bake shop was shown a picture of Rolito Go and he positively identified him as the same person who had shot Maguan. Having established that the assailant was probably Rolito Go, the police launched a manhunt for him. July 8, 1991, Rolito presented himself before the San Juan Police Station to verify news reports that he was being hunted by the police; he was accompanied by two (2) lawyers. The police detained him. An eyewitness to the shooting, who was at the police station at that time, positively identified him as the gunman.

Judge Pelayo however, motu proprio issued an Order, embodying the following: 1.Order which granted bail was recalled; Rolito was given 48 hours from receipt of the Order to surrender himself; 2.Order which granted leave to the prosecutor to conduct preliminary investigation was recalled and cancelled; 3.Rolitos omnibus motion for immediate release and preliminary investigation was treated as a petition for bail and set for hearing on 23 July 1991.

Complaint Filed by the Police: 1. Frustrated homicide. Filed at the Prosecutor of Rizal. Rolito Go was informed of this in the presence of his lawyers, and that he could avail himself of his right to preliminary investigation but that he must first sign a waiver of the provisions of Article 125 of the RPCe. Rolito Go refused to execute any such waiver. 2. Murder. While the complaint was still with the Prosecutor, and before an information could be filed in court, the victim, Eldon Maguan, died of his gunshot wounds. The Prosecutor, filed an information for murder instead of frustrated homicide at the RTC. No bail was recommended. At the bottom of the information, the Prosecutor certified that no preliminary investigation had been conducted because the accused did not execute and sign a waiver of the provisions of Article 125 of the RPC. Rolitos Action: Counsel for Rolito Go filed an Omnibus Motion for immediate release and proper preliminary investigation, alleging that the warrantless arrest of petitioner was unlawful and that no preliminary investigation had been conducted before the information was filed. Prayed for release on recognizance or on bail. Provincial Prosecutor Mauro Castro, acting on the omnibus motion, interposed that there is no objection over the grant of provisional liberty to Rolito Go upon cash bond of P100,000.00. Urgent ex-parte motion for special raffle - filed in order to expedite action on the Prosecutor's bail recommendation. The case was raffled to the sala of Judge Pelayo, who approved the cash bond posted by petitioner and ordered his release, Rolito was in fact released that same day. Prosecutors Action: Prosecutor filed with the RTC a motion for leave to conduct preliminary investigation and prayed that in the meantime all proceedings in the court be suspended. He stated that Rolito had filed before the Office of the Provincial Prosecutor of Rizal an omnibus motion for immediate release and preliminary investigation, which motion had been granted. RTC granted leave to conduct preliminary investigation and cancelling the arraignment set until after the prosecution shall have concluded its preliminary investigation.

Order of Events 1. First SC Petition: Rolito filed a Petition for certiorari, prohibition and mandamus assailing the Order promulgated by Judge Pelayo contending that the information was null and void because no preliminary investigation had been previously conducted, in violation of his right to due process. Rolito also moved for suspension of all proceedings in the case pending resolution by the Supreme Court of his petition; this motion was, however, denied by respondent Judge. 2. Surrender: July 23, 1991 3. Remand of Case to CA: July 24, 1991 4. Judge Pelayos Order: Regarding setting of Rolito Gos arraignment on August 23, 1991. 5. CA: Motion to restrain arraignment filed by Rolito Go on August 19, 1991 6. Judge Pelayos Commitment Order: August 23, 1991 - directing the Provincial Warden of Rizal to admit petitioner into his custody at the Rizal Provincial Jail. The same day, Rolito Go was arraigned. Hearings ensued. 7. Petition for Habeas Corpus in the CA: August 27, 1991. He alleged that in view of Judge Pelayos failure to join issues in the petition for certiorari earlier filed by him, after the lapse of more than a month, thus prolonging his detention, he was entitled to be released on habeas corpus. 8. CAs Decision. This is consolidated with the petition for certiorari, prohibition and mandamus. The CA then issued a resolution denying Rolito Gos motion to restrain his arraignment on the ground that that motion had become moot and academic. Trial commenced. CA rendered a consolidated Decision dismissing the two petitions, ratiocinating, thus: a. Petitioner's warrantless arrest was valid because the offense for which he was arrested and charged had been "freshly committed." b. Petitioner's act of posting bail constituted waiver of any irregularity attending his arrest. c. The trial court did not abuse its discretion because it had the inherent power to amend and control its processes so as to make them conformable to law and justice. d. Since there was a valid information for murder against petitioner and a valid commitment order the petition for habeas corpus could not be granted. 9. Counsel for Rolito filed Withdrawal of Appearance. Rolito filed this current Petition for Review of the CA

Page 31 of 33

Decision above. SC issued a Resolution directing Judge Pelayo to hold in abeyance the hearing of the criminal case until further orders. ISSUES: 1. Is there a lawful warrantless arrest by the San Juan Police in respect of Go? Theres none. 2. Has Go effectively waived his right to preliminary investigation? No. 3. Will Rolitos rights to preliminary investigation and bail be still available even if trial already commenced? Yes. HELD: 1. There was no lawful warrantless arrest of Rolito Go Contentions OSG: Petitioner had been validly arrested without warrant. Since petitioner's identity as the gunman who had shot Eldon Maguan on 2 July 1991 had been sufficiently established by police work, Go was validly arrested six (6) days later at the San Juan Police Station, invoking Nazareno v. Station Commander (a case where a majority of the Court upheld a warrantees arrest as valid although effected fourteen (14) days after the killing in connection with which Nazareno had been arrested). The provisions of Section 7, Rule 112 of the Rules of Court were applicable and because petitioner had declined to waive the provisions of Article 125 of the Revised Penal Code, the Prosecutor was legally justified in filing the information for murder even without preliminary investigation. ROLITO GO: The crime had not been "just committed" at the time that he was arrested, none of the police officers who arrested him had been an eyewitness to the shooting of Maguan and accordingly none had the "personal knowledge" required for the lawfulness of a warrantees arrest. Since there had been no lawful warrantless arrest, Section 7, Rule 112 of the Rules of Court which establishes the only exception to the right to preliminary investigation, could not apply. SC: The case does not fall within the terms of Section 5 of Rule 113 of the 1985 Rules on Criminal Procedure.4 The information upon which the police acted had been derived from statements made by alleged eyewitnesses to the shooting one stated that petitioner was the gunman; another was able to take down the alleged gunman's car's plate number which turned out to be registered in petitioner's
4

wife's name. That information did not, however, constitute "personal knowledge." There was no lawful warrantless arrest of Rolito Go. Section 7 of Rule 112 is also not applicable, because Rolito Go was not arrested at all, as he in fact placed himself at the disposal of the police authorities. He did not state that he was "surrendering" himself, in all probability to avoid the implication he was admitting that he had slain Eldon Maguan or that he was otherwise guilty of a crime. When the police filed a complaint for frustrated homicide with the Prosecutor, the latter should have immediately scheduled a preliminary investigation to determine whether there was probable cause for charging Rolito Go in court for the killing of Eldon Maguan. 2. Rolito Go did not waive his right to preliminary investigation OSG: Contends that the omnibus motion should have been filed with the trial court and not with the Prosecutor, and that the petitioner should accordingly be held to have waived his right to preliminary investigation. SC: The preliminary investigation was to be conducted by the Prosecutor, not by the Regional Trial Court. It is true that at the time of filing of petitioner's omnibus motion, the information for murder had already been filed with the Regional Trial Court: it is not clear from the record whether petitioner was aware of this fact at the time his omnibus motion was actually filed with the Prosecutor. Citing Crespo v. Mogul, SC says that the preliminary investigation conducted by the fiscal for the purpose of determining whether a prima facie case exists to warranting the prosecution of the accused is terminated upon the filing of the information in the proper court. The rule therefore is that once a complaint or information is filed in Court any disposition of the case such as its dismissal or the conviction or acquittal of the accused rests in the sound discretion of the Court. Although the fiscal retains the direction and control of the prosecution of criminal cases even while the case is already in Court he cannot impose his opinion on the trial court. The Court is the best and sole judge on what to do with the case before it. Court believes that Rolito Go did not waive his right to a preliminary investigation. The right to have a preliminary investigation conducted before being bound over to trial for a criminal offense and hence formally at risk of incarceration or some other penalty, is not a mere formal or technical right; it is a substantive right. The question may be raised whether petitioner still retains his right to a preliminary investigation in the instant case considering that he was already arraigned on 23 August 1991. RULE: the right to preliminary investigation is waived when the accused fails to invoke it before or at the time of entering a plea at arraignment AS REGARDS BAIL:

Sec. 5 Arrest without warrant; when lawful. A peace officer or a private person may, without warrant, arrest a person: (a) When, in his presence, the person to be arrested has committed, is actually committing, or is attempting to commit an offense; (b) When an offense has in fact just been committed, and he has personal knowledge of facts indicating that the person to be arrested has committed it; (c) When the person to be arrested is a prisoner who has escaped from a penal establishment or place where he is serving final judgment or temporarily confined while his case is pending, or has escaped while being transferred from one confinement to another. In cases falling under paragraphs (a) and (b) hereof, the person arrested without a warrant shall be forthwith delivered to the nearest police station or jail, and he shall be proceed against in accordance with Rule 112, Section 7.

Page 32 of 33

Posting bail does not effect a waiver of right to preliminary investigation Go asked for release on recognizance or on bail and for preliminary investigation in one omnibus motion. He had thus claimed his right to preliminary investigation before Judge Pelayo approved the cash bond posted by him and ordered his release. However, the failure to accord preliminary investigation, while constituting a denial of the appropriate and full measure of the statutory process of criminal justice, did not impair the validity of the information for murder nor affect the jurisdiction of the trial court. The Prosecutor had actually agreed that Rolito Go was entitled to bail.

3. Impact of commenced trial Rolito still entitled to preliminary investigation, and bail. These are matters of rights which cannot be withheld from him. Disposition: Petition granted. The Office of the Provincial Prosecutor is ordered to conduct a preliminary investigation of the charge of murder against petitioner Go. The trial on the merits of the criminal case in the RTC shall be suspended to await the conclusion of the preliminary investigation. Rolito is ordered released upon posting of cash bail bond of One Hundred Thousand Pesos (P100,000.00), without prejudice to any lawful order that the trial court may issue. SEPARATE OPINIONS GUTIERREZ, JR., J., concurring: Concurs with the majority but is at a loss for reasons why an experienced Judge as the respondent in this case insist on proceeding to trial in a sensational murder case without preliminary investigation. CRUZ, J., concurring: Thought first that Rolito Go waived his right to preliminary investigation. However from the transcripts, it was proved that he consistently asked for a preliminary investigation. The trial court has been moved by a desire to cater to public opinion to the detriment of the impartial administration of justice. Rolito Go as portrayed by the media is not exactly a popular person, the trial court should not have been influenced by this irrelevant consideration, remembering instead that its only guide was the mandate of the law. GRIO-AQUINO, J., dissenting: There is no need to conduct a preliminary investigation given that the prosecution has already presented four witnesses. Believes that the conduct of a preliminary investigation would be supererogatory. It should be remembered that as important as is the right of the accused to a preliminary investigation, it is not a constitutional right. Its absence is not a ground to quash the information (Doromal vs. Sandiganbayan). It does not affect the court's jurisdiction, nor impair the validity of the information (Rodis vs. Sandiganbayan), nor

constitute an infringement of the right of the accused to confront witnesses (Bustos vs. Lucero). The motion for a preliminary investigation is not more important than his application for release on bail, just as the conduct of such preliminary investigation is not more important than the hearing of the application for bail. The court's hearing of the application for bail should not be subordinated to the preliminary investigation of the charge. The hearing should not be suspended, but should be allowed to proceed for it will accomplish a double purpose. The bail hearing may not be suspended because upon the filing of an application for bail by one accused of a capital offense, "the judge is under a legal obligation to receive evidence with the view of determining whether evidence of guilt is so strong as to warrant denial of bond." Takes exception to the statements in the ponencia that the "petitioner was not arrested at all" and that "petitioner had not been arrested, with or without a warrant. Arrest is the taking of the person into the custody in order that he may be bound to answer for the commission of an offense (Sec. 1, Rule 113, Rules of Court). An arrest is made by an actual restraint of the person to be arrested, or by his submission to the custody of the person making the arrest (Sec. 2, Rule 113, Rules of Court). When Go walked into the San Juan Police Station, and placed himself at the disposal of the police authorities who clamped him in jail after he was identified by an eyewitness as the person who shot Maguan, he was actually and effectively arrested. His filing of a petition to be released on bail was a waiver of any irregularity attending his arrest and estops him from questioning its validity. Votes to dismiss the petition and affirm the trial court's order. LARRANAGA V. CA March 13, 1998 / J. Puno

*listed under bail but the case doesnt say much about bail sabaw lang FACTS: Francisco Larranaga was charged with 2 counts of kidnapping and serious illegal detention and was detained at Bagong Buhay Rehabilitation Center His mother filed with the SC a petition for certiorari, prohibition and mandamus with writs of preliminary and mandatory injunction They allege that Larranaga was denied the right to preliminary investigation (PI) and wants to annul the information. His mom also filed a petition for habeas corpus OR bail SC has already issued a resolution that Larranaga was indeed deprived of his right to PI when the City Prosecutor insisted that he was only entitled to an inquest The judge refused to comply with SC resolution saying that Larranaga was already arraigned and he waived his right to PI

Page 33 of 33

Judge alleges that Larranagas counsels deliberately withheld from SC the omnibus order, supplemental order, and order of arraignment he issued that is why the SC was misled in its resolution Prosecution alleges the ff Larranaga was charged with a continuing offense so his arrest and detention 2 months after abduction of the victims was lawful Larranaga was arrested without a warrant so he comes within the purview of Sec 7 of Rule 112 and not under Sec 3 Filing of the informations in court and the issuance of corresponding warrants of arrest by another judge cured whatever defect there was in Larranagas arrest and detention Larranaga was validly arraigned and the validity of such arraignment was not set aside Case of Sanchez v. Demetriou applies Larranaga is no longer a minor

PI is part of procedural due process it cannot be waived unless the waiver appears to be clear and informed

(2) Larranaga shouldnt be released from detention Sanchez v. Demetriou says that the filing of charges and issuance of the warrant of arrest against a person invalidly detained will cure the defect of that detention or at least deny him the right to be released because of such defect The absence of a PI will not justify Larranagas release because such defect will not nullify the information and the warrant of arrest against him

ISSUE: (1) WON Larranaga is entitled to a PI (2) WON he should be released from detention pending PI HELD: (1) yes (2) no RATIO: (1) Larranaga is entitled to a PI Sec 7 of Rule 112 only applies to persons lawfully arrested without a warrant Larranaga wasnt arrested by a peace officer or a private person PNP CIG went to his school in QC without a warrant and Larranaga resisted the arrest and called his lawyer His lawyer questioned the legality of the warrantless arrest before the CIG Legal Officer and so the PNP was ordered to stop the arrest and allowed Larranaga to go home There was no restraint upon Larranagas person and so Sec 7 of Rule 112 doesnt apple SC rejects prosecutors argument that Larranaga was actually committing a crime at the time of arrest since kidnapping with serious illegal detention is a continuing crime There is no showing that at the time of arrest, Larranaga was detaining the victim There was no waiver of PI. Mere failure to appear before the City Prosecutor cannot be construed as a waiver of his right to PI Right to PI is waived when the accused fails to invoke it before or at the time of entering a plea at arraignment Larranaga actively and consistently demanded for a PI before he was charged in court

Vous aimerez peut-être aussi